internal medicine topics

73
Internal Medicine exam questions; 3 rd dep. 3 rd year, 2 nd semester 08/09 Exam Questions - 3rd Year, 2nd Semester Academic Year 2008/2009 Tests of the anterior pituitary function. Hypopituitarism. Syndromes of anterior pituitary hyperfunction. Disorders of the posterior pituitary. Diabetes mellitus. Hypoglycemic disorders. Pancreatic islet Diseases of the hypothalamus. cell tumors. Evaluation of the adrenal function. Adrenocortical hypofunction. Cushing's syndrome. Mineralocorticoid excess states. The adrenogenital syndromes. The disorders of the adrenal medulla. Tests of thyroid function and thyroid regulation. Hyperthyreoidism. Hypothyroidism. Thyroiditis. Sporadic and endemic goiter. Benign and malignant tumors of the Hyperparathyroidism. thyroid. Hypoparathyroidism. Diseases of the breast. The carcinoid syndrome. The disorders of lipid metabolism. Principles of alimentation and hyperalimentation. Protein-calory undernutrition. Anorexia nervosa and bulimia nervosa. Obesity. Disorders of purine metabolism. Gout. Alcoholism. Amenorrhea. Hypogonadism in males. Hirsutism. Acid-base disorders. ECH 1

Upload: bogdan-urichianu

Post on 02-Dec-2015

224 views

Category:

Documents


1 download

DESCRIPTION

internal medicine topicsadthat u can see as a student

TRANSCRIPT

Page 1: Internal Medicine Topics

Internal Medicine exam questions; 3 rd dep. 3 rd year, 2 nd semester 08/09

Exam Questions - 3rd Year, 2nd Semester

Academic Year 2008/2009

Tests of the anterior pituitary function. Hypopituitarism.

Syndromes of anterior pituitary hyperfunction. Disorders of the posterior pituitary.

Diabetes mellitus. Hypoglycemic disorders. Pancreatic islet

Diseases of the hypothalamus. cell tumors.

Evaluation of the adrenal function. Adrenocortical hypofunction.

Cushing's syndrome. Mineralocorticoid excess states.

The adrenogenital syndromes. The disorders of the adrenal medulla.

Tests of thyroid function and thyroid regulation. Hyperthyreoidism.

Hypothyroidism. Thyroiditis.

Sporadic and endemic goiter. Benign and malignant tumors of the

Hyperparathyroidism. thyroid.

Hypoparathyroidism. Diseases of the breast.

The carcinoid syndrome. The disorders of lipid metabolism.

Principles of alimentation and hyperalimentation. Protein-calory undernutrition.

Anorexia nervosa and bulimia nervosa. Obesity.

Disorders of purine metabolism. Gout. Alcoholism.

Amenorrhea. Hypogonadism in males.

Hirsutism. Acid-base disorders.

Volume disorders. Osmolality disorders.

Vitamine deficiencies and hypervitaminoses.

Budapest, 17. January, 2009. Dr. Katalin Keltai

English Tutor

ECH 1

Page 2: Internal Medicine Topics

Tests of the anterior pituitary function

-Basal pituitary hormones: ACTH; TSH; LH, FSH; GH; PRL-Target gland hormones: cortisol (circadian); thyroxine, T3; testosterone or estradiol; IGF-1

Stimulation tests-stimulation of the hypothalamus:

-INS hypoglycemia test stimulates ACTH, GH, PRL synthesis. Contraindicated in patients >65yrs, pt.’s with CHD, uncontrolled HTN or epilepsy

-clonidine/arginine: stim. release of GH-clomiphene: test LH, FSH. It inhibits the action of estrogen on the

hypothalamus by binding to estrogen Rs for a long time prevents normal R recycling, in hypothalamic R no loss of FB FSH & LH

-metoclopramide: test PRL. It binds to DA D2-Rs antagonistically inhibits DA PRL. Contraindicated in pheochromocytoma-stimulation of the ant. pit. with hypophysiotropic hormones: CRH; TRH; GnRH, GHRH; TRH (theoretical, rarely of practical clinical significance)-stimulation of the target gland with tropic hormones:

-Short Synacthen test (short ACTH stim. test): to assess adrenal axis: do plasma cortisol before and 30 min after tetracosactide (=Synacthen) 250 g IM. Addison’s is excluded if 2nd cortisol >550 nmol/L

-hCG test: LH, FSH

Other dynamic tests:-INS tolerance test (ITT): in specialist centers, to assess adrenal and GH axes. Contraindications: epilepsy, heart disease, adrenal failure. Done in the morning by injecting IV INS to induce hypoglycemia, causing stress to ACTH and GH secretion. Glc must fall below 2.2 mM, normal is GH >20 mU/L and peak cortisol >550 nmol/L-when ITT is contraindicated: arginine + GHRH test; glucagon stimulation test-glucose tolerance test (OGTT): with GH measurement; done if acromegaly is suspected; test is done at 09.00 with fasting from midnight. GH and glc samples taken at 0, 30, 60, 90, 120, 150 min. Normally GH secretion is inhibited by a rice in glc, and GH should be undetectable – in acromegaly there is failure to suppress GH release (false positive results are seen in puberty, pregnancy, hepatic and renal disease, anorexia nervosa, and DM)-dexamethasone suppression test: screening test to diagnose and differentiate among types of Cushing’s and other hypercortisol states. Give dexamethasone 1mg PO at midnight; check serum cortisol before, and at 8AM. In normal pt.’s this high dose of steroid causes neg. FB ACTH and cortisol secretion to <50 nmol/L; in Cushing’ syndrome, there is a failure to suppress cortisol secretion

ACTH Cortisol InterpretationUndetectable/ Not suppressed Adrenal Cushing’s sy. is likely

/ Not suppressed Ectopic ACTH prod. is likely

/ SuppressedPituitary Cushing’s d. should be considered (do MRI)

Hypopituitarism

ECH 2

Page 3: Internal Medicine Topics

-diminished secretion of ant. pituitary hormones (loss or absence of >75% of anterior pituitary parenchyma-affected in the following order: GH, gonadotropins (FSH, LH), PRL, TSH, ACTH-panhypopituitarism is usually caused by irradiation, surgery or pituitary tumor-hypopituitarism accompanied by posterior pituitary dysfunction (diabetes insipidus) is almost always of hypothalamic origin

Causes are from 3 levels:-Hypothalamus: Kallman’s sy. (gonadotropin releasing hormone def., often with anosmia and color blindness secondary hypogonadism), tumor, inflammation, infection (meningitis, TB), ischemia-Pituitary stalk: trauma, surgery, mass lesion (craniopharyngioma –from Rathke’s pouch, commonest childhood cranial tumor), meningioma, carotid artery aneurysm-Pituitary: tumor, irradiation, inflammation, infiltration (hemochromatosis, amyloidosis, metastatic cancer), ischemia (apoplexy, Sheehan sy. – postpartum pituitary necrosis)

Features:-GH lack: central obesity; atherosclerosis; dry wrinkly skin; dec. strength, balance, CO; osteoporosis; dec. glc; growth failure (pituitary dwarfism) in children-gonadotropin lack: ♀: oligomenorrhea/amenorrhea, osteoporosis, breast atrophy, dyspareunia, dec. fertility and libido. ♂: erectile dysfunction, dec. libido and muscle bulk, hypogonadism-thyroid lack: tiredness, lethargy, depression, cold intolerance, weight gain, constipation, menorrhagia, hoarse voice, poor cognition/dementia, myalgia-corticotropin lack: nonspecific symptoms: fatigue, weakness, anorexia etc.-prolactin lack: rare – failure of lactation-pallor from loss of stimulatory effects of MSH on melanocytes-If the cause is pituitary tumor mass effect, or hormone secretion with dec. secretion of other hormones (eg. prolactinoma, acromegaly, Cushing’s)

Tests:-Basal:

LH & FSH or Testosterone or estradiol TSH or T4 Prolactin May be due to loss of DA from hypoth.IGF-1 cortisol

Na+ - dilutionHb (normochromic, normocytic)-Dynamic tests:

-Short Synacthen test: ACTH stim. test, to assess the adrenal axis-ITT: assess adrenal and GH axes; IV INS hypoglycemia ( cortisol

and GH secretion)

ECH 3

Page 4: Internal Medicine Topics

-Arginine + GH releasing hormone test-Glucagon stim. test

-MRI to look for lesion

Treatment:-hormone replacement, treatment of underlying cause-Hydrocortisone for secondary adrenal failure-Thyroxine for hypothyroid- For symptoms of hypogonadism and to prevent osteoporosis: ♀estrogen (estradiol implants, patches, pills) ♂ testosterone enanthate (IM, gels, tablets)-Gonadotropin therapy to induce fertility(-Somatropin for GH def.)

!Hypopituitary coma: usually develops gradually in a person with known hypopituitarism (rarely due to infarction of pituitary tumor – pituitary apoplexy)

-symptoms: headache, meningism (DDx: subarachnoid hemorrhage)-pt present with headache, ophthalmoplegia, consciousness,

hypotension, hypothermia, hypoglycemia-tests: T4, cortisol, TSH, ACTH, glc. Pituitary fossa CT/MRI-Rx: hydrocortisone sodium succinate + T3 (after hydrocortisone);

surgery if the cause is pituitary apoplexy

Syndromes of anterior pituitary hyperfunction

-adenohypophysis; originates from invagination of oral ectoderm (Rathke’s pouch); peptide hormones; pars distalis, tuberalis, intermedia; ACTH, TSH, FSH, LH, GH, PRL, -endorphin-10% of intracranial neoplasms (almost always benign adenomas); peak incidence from 30s to 50s

-chromophobe: 70%, PRL (ACTH, GH); local pressure effect in 30%-acidophil: 15%, GH, PRL; local pressure effect in 10%-basophil: 15%, ACTH; local pressure effect rare-local pressure effects: headache, bilateral temporal hemianopsia, CN

palsies, disturbance of hypothalamic centers of T, sleep, and appetite, CSF rhinorrhea (erosion through floor of sella)-investigations: MRI, assessment of visual fields, screening tests (PRL, IGF-1, ACTH, cortisol, TFTs, LH/FSH, testosterone in males, short Synachten test, OGTT (acromegaly), water deprivation test (DI)

-water deprivation test: to see if the kidneys persist in producing dilute urine despite dehydration-treatment: surgery (trans-sphenoidal, trans-frontal), post-op radiotherapy if complete removal has not been possible-pituitary apoplexy: acute hemorrhage into an adenoma rapid enlargement of the lesion, depression of consciousness; may cause sudden death (Rx: steroids)-functional (hormone excess, clinical manifestation of hyperproduction) vs. silent (immunohistochemical demonstration of hormone production at tissue level only; without clinical manifestation of hormone excess)

ECH 4

Page 5: Internal Medicine Topics

-approx. 40% of somatotropic adenomas and a minority of corticotropic adenomas bear GNAS1 mutations (chr. 20q13; mutation in -subunit of Gs GTPase activity, constitutive activation of Gs, persistent activation of cAMP, unchecked cellular proliferation)-cellular monomorphism, absence of reticulin network

Prolactinoma with hyperprolactinemia-most common pituitary tumor (30%)-PRL >5000 mU/L is likely to be due to a prolactinoma (N=♂<450 mU/L, ♀<600 mU/L (<900 pmol/L))

-chromophobic staining-hyperprolactinemia can be caused by hypothalamic lesions or by medications (methyldopa, reserpine) that interfere with dopamine (prolactin-inhibitory factor) secretion. Can be associated with estrogen therapy-DDx: pregnancy, high-dose estrogen therapy, renal failure, hypothyroidism (TRH), stalk effect (mass in suprasellar compartment disturb normal inhibitory influence of hypothalamus on prolactin secretion)-loss of libido, infertility, amenorrhea, galactorrhea in women-loss of libido, infertility, impotence, sometimes galactorrhea in men-management: dopamine agonists (bromocriptine, cabergoline)

Somatotropic adenoma with hypersecretion of GH-2nd most common pituitary tumor, presents bw. 30-50 yrs old(-hypersecretion of GH is rarely due to ectopic production from carcinoid tumor)-acidophilic staining (old name: acidophilic adenoma)-secondary hyperproduction of somatomedins by the liver. Effects are caused by GH and somatomedins (IGF-1, somatomedin C)-if adenoma develops before epiphyseal closure: gigantism-after epiphyseal closure: overgrowth of the jaws (+prognathism), face, hands, feet, macroglossia, generalized enlargement of viscera, hyperglycemia (DM), osteoporosis, HTN-the adenoma can cause local compression effects due to expansion of the tumor within the sella turcica-complications: IGT, DM; BP, LV hypertrophy, cardiomyopathy-Rx: somatostatin analogues (octreotide, lanreotide)

Corticotropic adenoma and hypersecretion of ACTH increased production of adrenal cortical hormones (hypercorticism) (PAS+)-hyperpigmentation (ACTH is synthesized as part of a larger prohormone with MSH)-Cushing disease:

-hypercorticism due to a corticotropic adenoma of the pituitary (basophilic adenoma)

-hypercorticism associated with very small, sometimes multiple pituitary adenomas (<1 cm diameter; basophilic microadenomas)-Cushing syndrome:

-hypercorticism regardless of cause (most often pituitary, less often of adrenal origin)

-ectopic ACTH production by various tumors (SCC of the lung!)-Nelson syndrome:

ECH 5

Page 6: Internal Medicine Topics

-large, clinically aggressive corticotropic adenoma developed after surgical removal of adrenals (for treatment of Cushing syndrome)

-caused by loss of inhibitory effect of adrenal corticosteroids on a preexisting microadenoma

-hypercortisolism does not develop (adrenals are absent) mass effect

-Gonadotropic (LH- and FSH-producing) adenoma: usually do not cause a recognizable clinical syndrome-Thyrotropic (TSH-producing) adenoma: about 1% of all pituitary adenomas; rare cause of hyperthyroidism-Nonfunctioning pituitary adenomas: silent or true hormone-negative adenomas (most often chromophobic); many have been reclassified using improved diagnostic techniques; nonfunctioning adenomas constitute approx. 25% of all pituitary adenomas, typical presentation is mass effect (bilateral hemianopsia, CN III, IV, VI palsies). May compromise the residual anterior pituitary sufficiently to produce hypopituitarism-Pituitary carcinoma: rare; virtually always have distant metastases

Disorders of the posterior pituitary

-neurohypophysis; pars nervosa (posterior lobe), infundibulum, (median eminence); storage and secretion of OT, ADH (VP, AVP)-abnormal oxytocin synthesis and release has not been associated with significant clinical abnormalities-ADH deficiency: central diabetes insipidus: polyuria (>3 L/day; inability of kidney to resorb water from the urine)

-excretion of large volumes of dilute urine with low specific gravity-serum sodium and osmolality polydipsia, dehydration-DDx: head trauma, neoplasms, inflammatory disorders of hypothalamus

and pituitary, surgical procedures involving hypoth. or pit.-Rx: desmopressin

-ADH excess: SIADH (syndrome of inappropriate ADH secretion): resorption of excessive amounts of free water hyponatremia

-cerebral edema neurologic dysfunction. Total body water is , but blood volume remains normal and peripheral edema does not develop

-DDx: SIADH is most commonly caused by ectopic ADH production by malignant neoplasm (SCC of the lung), non-neoplastic diseases of the lung, local injury to hypothalamus and/or posterior pituitary

Diabetes Mellitus

-measurement of glc: blood (venous blood, venous plasma, capillary blood), urine, CSF

ECH 6

Page 7: Internal Medicine Topics

-in presence of cells, bacteria-concentration in whole blood is 15% lower than that in plasma (RBCs

contain more dry material)-venous blood concentration is about 0.5-1 mM lower than that of the

capillary if INS concentration is high (due to glc uptake of tissues)-enzymatic methods: hexokinase, glc-oxidase; paper strip methods: glc-

oxidase (SMBG); reducing methods: not specific for glc, used to detect mellituria in newborns

WHO diagnostic algorithm (Europe)-Step 1: random blood glc

<5.5 mM: normal carbohydrate metabolism5.5-11.0: fasting blood glc>11.0: classic symptoms? If yesDM, if nofasting blood glc

-Step 2: fasting blood glc<5.0 mM: normal carbohydrate metabolism5.0-6.0: regular checkup necessary6.0-7.0: OGTT>7.0: repeat test <7.0: OGTT, >7.0: DM

-Step 3: OGTT-fast overnight. Give 75 g glc in 300 mL water, measure venous plasma glc

before and 2h after the drink

-IGT: fasting plasma glc <7.0 mM and OGTT 2h >7.8 but <11.0-IFG: fasting plasma glc <6.1 mM but <7.0 (do OGTT!)-measurement of glycated Hb (HbA1c, GHB): to check treatment of diabetics, twice/year; reflects mean glc concentration of the past 8 weeks, treatment goal is <7%

Type 1 (IDDM, juvenile or ketosis-prone DM)-begins early in life, less common than type 2-failure of INS synthesis

ECH 7

Page 8: Internal Medicine Topics

-genetic predisposition compl. by AI inflammation of islets triggered by viral infection or environmental factors

-incidence greatly in pt’s with point mutation in HLA-DQ gene. Markedly in HLA-DR3- and HLA-DR4-positive individuals-islets are small, -cells greatly in no., insulitis marked by lymphocytic infiltration-carbohydrate intolerance with hyperglycemia polyuria, polydipsia, weight loss despite increased appetite, ketoacidosis, coma, death

-ketoacidosis: catabolism of fat KB production-detection of KB (acetoacetic acid): nitroprusside sodium (Legal test) –

tablet or strip form, qualitative or semiquantitative result-latent AI diabetes of adults (LADA) is a form of type 1, with slower progression to INS dependence in later life

Type 2 (NIDDM, adult-onset, or ketosis-resistant DM)-often begins in middle age, more common than type 1-positive family history is more frequent than in type 1-often associated with mild to moderate obesity, lack of exercise and calorie excess. Typically progresses from IGT/IFG-increased INS resistance due to decreased cell membrane INS receptors or postreceptor dysfunction. Also associated to impaired processing of proINS to INS, decreased sensing of glc by -cells or impaired function of IC carrier proteins-causes of INS resistance: obesity, pregnancy, renal failure, polycystic ovarian syndrome, acromegaly, Asian descendance, CF, Werner’s syndrome (adult progeria), TB drugs, Cushing’s, metabolic syndrome (syndrome x)

-obesity may cause INS resistance by rate of release of non-esterified FAs causing post-receptor defects in INS action-focal islet fibrosis and hyalinization (amylin deposits); amylin (islet amyloid polypeptide/IAPP) deposition in islets is characteristic for type 2, interfere with proINSINS conversion or glc sensing-plasma INS is /-mild carbohydrate intolerance (diet, oral antidiabetic agents)-maturity-onset DM of the young (MODY) is a rare AD form of type 2, causing mild hyperglycemia and hyposecretion of INS (no loss of -cells); earlier onset, caused by diverse group of single gene defects (family history)

Type 1 DM Type 2 DMEpidemiology Younger patients Older patientsGenetics HLA-DR3 and –DR4 linked No HLA associationEtiology AI -cell destruction INS resistance, -cell dysfunctionPresentation Polydipsia, polyuria, weight loss,

ketoacidosisOften asymptomatic; presents with micro- or macrovascular complications

Autoantibodies: islet cell antibodies (ICA), anti-glutamic acid decarboxylase (GAD) Ab.

Secondary DM-pancreatic disease

-surgery: >90% removed; trauma; CF

ECH 8

Page 9: Internal Medicine Topics

-hereditary hemochromatosis (bronze diabetes): excess iron absorption, parenchymal deposition of hemosiderin reactive fibrosis in pancreas (, liver, heart)

-pancreatitis: acute pancreatitis is characterized by hyperglycemia; chronic pancreatitis may result in islet cell destruction

-carcinoma of the pancreas: DM may be presenting sign-Cushing sy.: hyperglycemia due to increased gluconeogenesis and impaired peripheral utilization of glucose-acromegaly: hyperglycemia due to anti-insulin-like effect of GH-glucagon hypersecretion: promotes glycogenolysis (islet -cell tumor: glucagonoma)-pheochromocytoma and hyperthyroidism are sometimes associated with hyperglycemia-others: acanthosis nigricans: congenital lipodystrophy with INS receptor Ab.’s; GSD-pregnancy

-gestational DM (transient): infant may be born with hyperplasia of pancreatic islets, hypoglycemia

-DM is associated with increased fetal birth weight, increased fetal mortality (RDS/hyaline membrane disease)

Kidney damage in DM-increased width of glomerular BM (earliest, most common finding)-diffuse diabetic glomerulosclerosis, nodular diabetic glomerulosclerosis (Kimmelstiel-Wilson d.), arteriolar lesions, exudative lesions (fibrin cap, capsular drop)-Armanni-Ebstein lesion: tubular deposition of glycogen; uncommon result of prolonged untreated hypoglycemia-pyelonephritis renal papillary necrosis-Rx: if microalbuminuria or proteinuria is present: ACE-I or AT2-blocker

Cardiovascular system damage in DM-atherosclerosis, ischemic heart disease, coronary artery disease-MI (3-5x increased risk) and peripheral vascular insufficiency (gangrene of lower extremities)-capillary BM thickening in multiple organs due to nonenzymatic glycosylation of membrane protein-Rx: statin (if evidence of IHD, peripheral or cerebrovascular d.), fibrates (TAG, HDL)

Eye damage in DM-cataract formation-proliferative retinopathy (retinal exudates, edema, hemorrhages, microaneurysms) blindness

-capillary endothelial change vascular leak microaneurysms capillary occlusion local hypoxia + ischemia new vessel formation

-high retinal BF caused by hyperglycemia (/BP/pregnancy) triggers new vessel formation, causing capillary pericyte damage. New vessels form on the

ECH 9

Page 10: Internal Medicine Topics

disc or ischemic areas, proliferate, bleed, fibrose, and can detach the retina; aspirin may prevent it

Nervous system damage in DM-peripheral neuropathy, changes in brain and SC-sensory neuropathy: symmetric sensory polyneuropathy – distal numbness (esp. vibrational sense is lost), tingling, pain-mononeuritis multiplex: esp. III & VI CN-amyotrophy: painful wasting of quadriceps (+ other pelvifemoral muscles)-autonomic neuropathy: postural BP drop, cerebrovascular autoregulation, gastroparesis, urine retention, erectile dysfunction, diarrhea

Other changes in DM-liver: fatty change-skin: xanthomas, furuncles and abscesses because of propensity to infection (frequent fungal infections, esp. with Candida)

Treatment, monitoring etc.: -monitor capillary glc measurements, exercise, diet (saturated fats, sugar, starch-carbohydrate, moderate protein), smoking cessation, foot-care; care during pregnancy, advise to inform driving licence authority (loss of awareness of hypoglycemia may lead to loss of driving licence)-intercurrent illness (eg. influenza) often INS requirements despite reduced food intake, blood glc should be checked >4 times/day-type 1 DM patients: INS, patients self-adjust dose in the light of exercise and calorie intake. Design the INS regimen to suit your pt’s lifestyle, not vice versa

-SC INS: 100 U/mL; ultra-fast acting (at start of meal/immediately after), soluble INS (15-30 min before meal), intermed., long-acting, long-acting analogue, pre-mixed INS (ultra-fast component + soluble INS)

-INS glargine: long-acting recombinant human INS analogue used one time daily at bedtime. Soluble at acidic pH, precipitates in subcutaneous tissue and is slowly released from a depot (INS detemir has similar characteristics)

-inhaled INS is being developed (rapid-acting)-type 2 DM patients: diet + exercise (promote INS sensitivity); oral hypoglycemic agents: metformin, metformin + sulfonylurea (/glitazone, etc.); INS

!Diabetic ketoacidosis:-hyperglycemic ketoacidotic coma only occurs in type 1 DM. Precipitants include infection, surgery, MI, non-compliance, or wrong INS dose. Dx requires ketosis and acidosis (pH<7.3)-signs & symptoms: polyuria, polydipsia, lethargy, anorexia, hyperventilation, ketotic breath, dehydration, vomiting, abd. pain, coma-tests: lab glucose, urea/electrolytes/creatinine in plasma, HCO3

-, amylase, osmolality, art. blood gas, CBC, blood cultures; urine testing for ketones; chest X-ray

-glc is usually , high WBC count may be seen in the absence of inf., often no fever is inf. is present, hyponatremia is common (osmolar compensation for hyperglycemia), serum amylase is often raised

ECH 10

Page 11: Internal Medicine Topics

-Rx: correction of dehydration takes precedence; fluid replacement and dextrose administration, INS; K+ replacement; monitor electrolyte levels; treat inf. if present; -complications: cerebral edema, aspiration pneumonia, hypokalemia, hypomagnesemia, hypophosphatemia, thromboembolism

!Hypoglycemic coma:-usually rapid onset, may be preceded by odd behavior (eg. aggression), sweating, pulse, seizures-Rx: 20-30g IV dextrose (200-300ml of 10% dextrose) – prompt recovery. Glucagon is nearly as rapid as dextrose but will not work in drunk patients. Once conscious, give sugary drinks/meal

!Hyperglycemic hyperosmolar non-ketotic coma:-only in pt.’s with type 2 DM; longer history (1wk) with marked dehydration and glc >35mM. Acidosis is absent since there has been no switch to KB formation. Osmolality >340mOsm/kg. Focal CNS signs may occur. High risk of DVT, give full heparin anticoagulation-Rx: rehydrate over 48h (0.9% saline IV), INS may not be needed. Look for underlying cause, eg. MI, bowel infarct

!Hyperlactemia is a rare but serious complication of DM (eg. after septicemia or biguanides use); blood lactate >5mM. Give O2, treat sepsis

Hypoglycemic disorders. Pancreatic islet cell tumors

-plasma glc <3 mM-Dx: Whipple triad: symptoms compatible with hypoglycemia, low blood glc level, symptoms disappear after increasing blood glc level-most common cause: INS effect in diabetic pt treated with INS: INS OD, delayed/missed meal, strong physical activity-other causes, fasting: alcohol, drug effect; pituitary insufficiency; severe liver disease, sepsis; Addison’s disease; insulinoma; non--cell tumors (secretion of INS-like substances by HCC, mesotheliomas, sarcomas. IGF-1 secretion by retroperitoneal fibrosarcomas and hemangiopericytomas)-reactive (postprandial) hypoglycemia: alimentary, after gastric surgery (dumping syndrome); inherited metabolic defect in children (eg. galactosemia, GSDs: von Gierke, Pompe, Cori)-ANS symptoms: cold sweat, tremor, anxiety, hunger, tachycardia, palpitations-CNS symptoms (neuroglycopenia): nervousness, altered behavior, drowsiness, disorientation, cramps, seizures, coma-Rx: oral sugar, long-acting starch (IV glc if pt cannot swallow); glucagon (short duration of effect: repeat after 20 min and follow with oral carbohydrate!)!Hypoglycemic coma: usually rapid onset; may be preceded by odd behavior (aggression), sweating, pulse, seizures. Rx: dextrose – prompt recovery. Glucagon is nearly as rapid as dextrose but will not work in drunk patients. Once conscious, give sugary drinks/meal

ECH 11

Page 12: Internal Medicine Topics

Insulinoma/-cell tumor-associated with MEN-1, usually benign-presents as fasting hypoglycemia (high or normal INS, no KB)

-screening test: hypoglycemia + plasma INS during a long fast-suppressive test: IV INS, measure C-peptide. Normally exogenous INS

suppresses C-peptide production, but not in insulinoma-Dx: CT/MRI + endoscopic pancreatic US (intra-operative visualization or US)-Treatment: surgical excision

Gastrinoma-often malignant, sometimes occur in extrapancreatic sites-gastrin hypersecretion, hypergastrinemia-associated with Zollinger-Ellison syndrome (marked gastric hypersecretion of HCl, recurrent peptic ulcer disease, hypergastrinemia)

Glucagonoma/-cell tumor-rare, lead to secondary DM-characteristic skin lesion: necrolytic migratory erythema

VIPoma-rare, secretes VIP-associated with Verner-Morrison syndrome/pancreatic cholera (watery diarrhea, hypokalemia, achlorhydria)

MEN I (Wermer syndrome)-hyperplasias or tumors of pituitary, parathyroid, pancreatic islets (thyroid, adrenal cortex), mutations in MEN I gene-pancreatic component can be Zollinger-Ellison syndrome, hyperinsulinism, or Verner-Morrison syndrome/pancreatic cholera

Diseases of the hypothalamus

-hypothalamic hormones: CRH, DA, GnRH, GHRH, SS, TRH [corticotropin-releasing hormone, dopamine, gonadotropin-releasing hormone, growth hormone-releasing hormone, somatostatin, thyrotropin-releasing hormone]

-hypothalamus controls body temperature, hunger, thirst, fatigue, and circadian cycles-due to the tight integration of the hypothalamus and the pituitary, and the relative inaccessibility of these glands, it is often difficult to tell if a condition is due to dysfunction in the hypothalamus or in the pituitary, though certain hormone tests can help distinguish between these causes

-Hypopituitarism can be originating from a hypothalamic level. Causes include Kallman’s sy. (gonadotropin releasing hormone def., often with anosmia and color blindness secondary hypogonadism), tumor, inflammation, infection (meningitis, TB), ischemia

Adiposogenital dystrophy/Babinski-Fröhlich syndrome/hypothalamic infantilism-obesity

ECH 12

Page 13: Internal Medicine Topics

-tumors of the hypothalamus appetite, secretion of gonadotropin-affect males mostly-characterized by feminine obesity, growth retardation, retarded sexual development, headaches, mental retardation, visual defects, polyuria and polydipsia

AR hypothalamic corticotropin deficiency-CRH (corticotropin-releasing hormone)-hypoglycemia, hepatitis

Tertiary adrenal insufficiency-cause: tumor of the hypothalamus (adenoma), antibodies, environment, head injury-all diagnostic lab results are low (CRH, ACTH, DHEA, DHEA-S, cortisol, aldosterone, rennin, sodium, potassium)

Neurogenic diabetes insipidus (central DI)-lack of ADH

Hypothalamic hamartoma-not detected on CT scans, isotense to gray matter on MRI images-1 out of 1-2 million individuals-gelastic (laughing) epilepsy, other seizures, precocious puberty, behavioral disorders

Pallister-Hall syndrome (hypothalamic hamartoblastoma syndrome)-hypothalamic hamartoma is characteristic: may lead to seizures or hormone abnormalities-usually presents with polydactyly, cutaneous syndactyly, bifid epiglottis, imperforate anus, kidney abnormalities-AD; mutations in GLI3 gene (protein product controls gene expression, mutated gene product can only repress target genes. GLI3 plays a role in the normal patterning of many organs and tissues before birth)

ROHHAD syndrome – rapid-onset obesity with hypothalamic dysfunction, hypoventilation and autonomic dysregulation-very rare, 35 cases worldwide-hypothalamic dysfunction: failed GH stimulation, polydipsia, hypernatremia, hyperprolactinema, hyperphagia, DI-resp. manifestations: alveolar hypoventilation, cardiorespiratory arrest, obstructive sleep apnea-autonomic dysregulation: ophthalmologic manifestations, thermal dysreg., GI dysmotility, altered perception of pain, altered sweating, cold hands and feet, tumor of neural crest origin-other symptoms: seizures, enuresis, hypotonia

Kallman's syndrome/hypothalamic hypogonadism-def. of GnRH (gonadotropin-releasing hormone) lack of LH and FSH-congenital anosmia (hyposmia)

ECH 13

Page 14: Internal Medicine Topics

-color blindness is common-delayed puberty (primary amenorrhea, lack of secondary sex characteristics in women; males may have micropenis)-untreated Kallman’s syndrome can lead to osteoporosis (do regular bone-density scans!)-GnRH neurons do not migrate properly from olfactory placode to hypothalamus during development; olfactory bulbs also fail to form/show hypoplasia

-XR (KAL1 gene), AD (KAL2), or AR (KAL3) trait. -Rx: hormone therapy: males are given hCG or testosterone, females are treated with estrogen and progestins. GnRH (LHRH) is administered to induce fertility

Shapiro syndrome-paroxysmal hypothermia (hypothalamic dysfunction of thermoregulation)-epilepsy, agenesis of corpus callosum

Evaluation of the adrenal function

-zona glomerulosa – mineralocorticoids (aldosterone) – control sodium and potassium balance-zona fasciculate – glucocorticoids (cortisol) – affect carbohydrate, lipid, and protein metabolism-zona reticularis – androgens (DHEA, DHEAS) – sex hormones, weak effect until peripheral conversion to testosterone/DHT-CRH from hypothalamus stimulates ACTH secretion from pituitary gland, which in turn stimulates cortisol and androgen production by adrenal cortex

-measure hormones in blood (total/free)-measure a hormone or its degradation products in the urine-stimulation tests-suppression tests-radiographic imaging studies: CT/MRI

-radioactive I-131 iodonorcholesterol uptake: detection of adrenal cortical tumors

-MIBG uptake: detection of pheochromocytomas

-saline loading test: DDx of tumor-derived hyperaldosteronism and essential hypertension: IV infusion of isotonic NaCl over a period of 2h, aldosterone and cortisol levels are measured during the loading. Expansion of the extracellular space reduces serum aldosterone levels in pt.’s who have essential HTN but does not affect those who have primary hyperaldosteronism. Furthermore, such pt.’s have much more aldosterone than cortisol in their blood

-lab tests useful for the Dx of Cushing’s syndrome:-blood cell counts (Hb, HCT, RBC count are high. Total WBC count is

normal, but there might be fewer than normal eosinophils and lymphocytes)-serum electrolytes (hypokalemic alkalosis might be present in pt.’s with

adrenal cc.)

ECH 14

Page 15: Internal Medicine Topics

-cortisol in serum (total/free; take several blood samples because of cyclic secretion – midnight free cortisol or cortisol in the saliva can also be measured)

-free cortisol in the urine (collected over 24h; best screening test)-low-dose dexamethasone suppression test (should suppress normal

morning rise in cortisol when injected IV at midnight: suppression does not occur in pt.’s with Cushing’s syndrome. Should be followed by a CRH stimulation test or high-dose dexamethasone suppression test for confirmation)

-high-dose dexamethasone suppression test suppresses below 50% increased levels of ACTH in pt.’s with pituitary adenoma, but does not suppress ACTH produced by malignant nonpituitary tumors

-ACTH in serum (in pt.’s with Cushing’s d. (pituitary adenoma) or ectopic ACTH production. in pt’s with adrenal tumors/hyperplasia, or pt.’s treated with steroids)

-lab tests of serum in Addison’s disease typically show Na+, Cl-, and HCO3-; K+,

dehydration, hypoglycemia, low serum aldosterone and cortisol, high ACTH-stim. of adrenal with ACTH gives negative results; primary adrenal

insufficiency can be distinguished from pituitary insufficiency by measuring aldosterone in the same blood sample: if the patient has pituitary insufficiency, aldosterone rises, but if the disease is caused by adrenal insufficiency, aldosterone does not rise

-Dx of pheochromocytoma:-lab testing: catecholamines and their derivatives are increased in blood

and urine (episodic release, collect urine over 24h. Metanephrine is the best screening test, but urinary free E or NE or VMA can also be measured)

-clonidine suppression test: clonidine is an 2-agonist and antihypertensive agent, it blocks the centrally regulated sympathomimetic nerves. Measurement of catecholamines in the blood and BP 3h after clonidine administration in normal ppl and those with essential HTN shows a suppressive effect, while in pt.’s with pheochromocytoma arterial pressure and blood catecholamines remain high

-radiologic localization of the tumor: CT/MRI; adrenal scanning with MIBG (131I-metaiodobenzylguanidine)

Adrenocortical hypofunction

-anyone on prednisolone for long enough to suppress the pituitary-adrenal axis or has overwhelming sepsis, or has metastatic cancer may suddenly develop adrenal insufficiency with deadly hypovolemic shock

Hypocorticism-primary (adrenal cause) vs. secondary (hypothalamic, pituitary dysfunction)-deficiency of glucocorticoids (cortisol) with associated mineralocorticoid (aldosterone) def.

-Addison’s disease (primary adrenocortical deficiency)

ECH 15

Page 16: Internal Medicine Topics

-most commonly idiopathic adrenal atrophy (AI lymphocytic adrenalitis)-formerly most commonly caused by TB, may also be caused by metastatic

tumor, and various infections-nonspecific symptoms: fatigue, weakness, anorexia, weight loss,

dizziness, fainting, myalgia, arthralgia, depression, psychosis, low self-esteem, nausea/vomiting, abdominal pain, diarrhea or constipation. “Think of Addison’s in all those with unexplained abdominal symptoms”

-hypotension; pigmentation of skin (due to ACTH); serum sodium, chloride, glucose, bicarbonate; serum potassium

-Dx: short Synacthen test-Rx: replace steroids (double steroids in febrile illness, injury or stress).

Yearly follow-up, good prognosis!Addisonian crisis: pt may present in shock (tachycardia, peripheral vasoconstriction, postural hypotension, oliguria, weak, confused, comatose) – usually a pt with known Addison’s d. (or someone on long-term steroids who has forgotten to take tablets). May present as hypoglycemia

-precipitated by infection, trauma, surgery-symptomatic treatment (monitor blood levels)

-Waterhouse-Friderichsen syndrome-catastrophic adrenal insufficiency and vascular collapse due to

hemorrhagic necrosis of the adrenal cortex-associated with DIC; caused by meningococcemia (meningococcal

meningitis)

-Secondary adrenocortical deficiency is almost always iatrogenic due to long term steroid therapy leading to suppression of the pituitary-adrenal axis. Other causes are rare and include hypothalamic-pituitary disease leading to ACTH production (mineralocorticoid production remains intact, no hyperpigmentation)

Mineralocorticoid excess states

-Primary hyperaldosteronism: excess production of aldosterone, independent of the rennin-angiotensin system sodium, water retention, renin release (negative FB of increased BP)

-pt’s are usually asymptomatic, may present with features of hypokalemia, weakness, cramps, paresthesia (sensation of tingling, pricking, or numbness of a person's skin), polyuria, polydipsia

-should be considered with the following features: HTN (refractory), hypokalemia and/or alkalosis on someone not on diuretics. Sodium tends to be mildly raised or normal-2/3 are due to a solitary aldosterone-producing adrenocortical adenoma (aldosteronoma) (Conn’s syndrome). 1/3 are due to bilateral adrenocortical hyperplasia (rare causes: adrenal carcinoma, glucocorticoid-remediable aldosteronism/GRA: ACTH regulatory element of 11-hydroxylase gene fuses to aldosterone synthase aldosterone production, under control of ACTH. Family history of early HTN))

ECH 16

Page 17: Internal Medicine Topics

-tests: U&E: do not rely on a low K+ (>20% are normokalemic); genetic tests are available for GRA

-DDx: renal artery stenosis is a more common cause of refractory HTN and K+

-treatment: Conn’s: laparascopic adrenalectomy. Hyperplasia: medically, spironolactone, amiloride etc. GRA: dexamethasone (if BP I still , spirolactone is an alternative). Adrenal cc.: surgery + post-op adrenolytic therapy (mitotane; poor prognosis)

-Secondary hyperaldosteronism: secondary to renal ischemia (renal artery stenosis), renal tumors, and edema (cirrhosis, nephritic syndrome, HF) stimulation of rennin-angiotensin system

-serum rennin is

-Bartter’s syndrome: AR salt wasting: sodium & chloride leak in the loop of Henle via a defective channel. Presents in childhood: failure to thrive, polyuria, polydipsia. Normal BP

-sodium loss volume depletion renin and aldosterone production hypokalemia, metabolic alkalosis, urinary K+ and Cl-

-Rx: K+ replacement, NSAIDS (inhibit PGs), ACE-i

Cushing’s syndrome

Hypercorticism: increased circulating glucocorticoids (cortisol)-exogenous corticosteroid medication (most common); corticotropic adenoma of pituitary (hypersecretion of ACTH, usually microadenoma); adrenal cortical adenoma or adrenal carcinoma; adrenal nodular hyperplasia; ectopic production of ACTH (SCC of the lung, carcinoid tumors) [rare: ectopic CRH production – medullary thyroid carcinoma, prostate carcinoma]

-Carney complex (a MEN syndrome, can cause adrenal hyperplasia) and McCune Albright syndrome (genetic disorder that can present with endocrine hyperfunction) may be rare ACTH-independent causes of Cushing’s syndrome

-tumors of adrenal cortex: adenoma is more common; hypercorticism cannot be suppressed by exogenous adrenal steroids in dexamethasone suppression tests (hypercorticism of pituitary origin can usually be suppressed)

-ACTH is in pituitary hypercorticism and in ectopic ACTH production (+ ectopic CRH prod.); it is low when hypercorticism is of adrenal origin (due to negative FB)-morphologic changes in adrenal gland: bilateral hyperplasia of zona fasciculata (when syndrome results from ACTH stimulation), adrenal cortical atrophy (when exogenous medication is the cause)-characteristics: redistribution of body fat: central obesity, moon face, dorsal buffalo hump, often with relatively thin extremities caused by muscle wasting; skin atrophy with easy bruising and purplish striae (esp. over abdomen); hirsutism. Other symptoms include muscle weakness, osteoporosis, amenorrhea, male impotence, hypertension, hyperglycemia (increased gluconeogenesis, impaired peripheral utilization of glc; IGT, DM), psychiatric dysfunction

ECH 17

Page 18: Internal Medicine Topics

-ectopic ACTH production usually don’t show classical features of Cushing’s. Symptoms: pigmentation (ACTH), hypokalemia, metabolic acidosis (cortisol leads to mineralocorticoid activity), weight loss, hyperglycemia-tests: random cortisol measurements are of no value (diurnal variation)

-overnight dexamethasone suppression test: 1mg dexamethasone PO at midnight, check serum cortisol before, and at 8AM. In normal pt: this high dose of steroid causes negative feedback, ACTH, cortisol. In Cushing’s: failure to suppress cortisol secretion

-24h urinary free cortisol: normal <280 nmol/24h-pseudocushing’s (false positive tests) are seen in depression, obesity,

alcohol excess, inducers of liver enzymes which rate of dexamethasone metabolism

-2nd line screening tests and localization tests-Rx: stop exogenous steroid medication; surgical excision (adrenalectomy); treated Cushing’s syndrome has a good prognosis-Remember Nelson’s syndrome: bilateral adrenalectomy removes negative FB development of a locally aggressive corticotropic adenoma in pituitary gland (indication for pituitary radiotherapy)

The adrenogenital syndromes

Adrenal virilism/congenital adrenal hyperplasia-adenoma, carcinoma or hyperplasia of zona reticularis-congenital (AR) enzyme defects diminished cortisol production, compensatory increased ACTH adrenal hyperplasia with androgenic steroid production

-21-hydroxylase def.: most common (95%), can result in salt loss & hypotension (vomiting and dehydration within first weeks of life)

-11-hydroxylase def.: much less common, results in salt retention & hypertension-may also be caused by tumors of the adrenal cortex-virilism (muscle bulk, body hair, deep voice) in females, precocious puberty in males-treatment: supply glucocorticoid (cortisol) to reduce hyperplasia and overproduction of androgens or mineralocorticoids (aldosterone)/replacement mineralocorticoid and salt/replacement testosterone or estrogen at puberty/additional treatments to optimize growth by delaying puberty or delaying bone maturation/genital reconstructive surgery

Late-onset congenital adrenal hyperplasia-androgen-secreting tumor of adrenal gland amenorrhea, clitoromegaly, deep voice, temporal hair recession, hirsutism

X—linked adrenal hypoplasia congenita-mainly affects males; XR; NR0B1 gene (Xp21.3-p21.2) deletion or mutation underdeveloped adrenals, hypothalamus, and pituitary-incomplete development of adrenal cortex

ECH 18

Page 19: Internal Medicine Topics

-adrenal insufficiency in infancy/childhood: vomiting, feeding difficulties, dehydration, hypoglycemia, shock

-affected males lack sex hormones underdeveloped reproductive tissues, cryptorchidism, delayed puberty, infertility

Disorders of the adrenal medulla

Tumors-Pheochromocytoma:

-arise from SY paraganglia cells = chromaffin cells (paraganglioma=derived from extra-adrenal chromaffin cells, usually by the aortic bifurcation)

-usually benign: 10% are malignant-hyperproduction of catecholamines (NE, E) by tumor paroxysmal HTN

(surgically correctable!)-can cause hyperglycemia, anxiety, chest tightness-complications: HF, dilated cardiomyopathy, arrhythmias, stroke and

death due to hypertensive crisis-90% are sporadic, but pheochromocytoma can also be part of MEN IIa or

MEN IIb (III). It is associated with neurofibromatosis or von Hippel-Lindau disease (hemangioblastomas in cerebellum, SC, retina)

-urinary excretion of catecholamines and their metabolites-tests: 3x24h urinary collections for catecholamines; CT/MRI; meta-

iodobenzylguanidine (chromaffin-seeking isotope) scan: useful for detection of extra-adrenal tumors

-lifelong follow-up post-op: malignant recurrence may present late!Pheochromocytoma emergency: stress, abdominal palpation, parturition, general anesthetic, contrast media may produce dangerous hypertensive crises (pallor, pulsating headache, HTN)

-Rx: phentolamine (repeat), phenoxybenzamine when BP is controlled. Surgery is usually done electively after 4-6wks to allow full -blockade and volume expansion

-Neuroblastoma:-highly malignant catecholamine-producing (“small blue cell”) tumor;

occurs in early childhood-urinary catecholamines and metabolites-HTN, large abdominal mass-tumor is characterized by amplification of N-myc oncogene (1000x of

copies/cell). Malignant cells of neuroblastoma sometimes differentiate into benign cells, reflected by a marked reduction in gene amplification

-occasionally the tumor converts into a more differentiated form = ganglioneuroma

Tests of thyroid function and thyroid regulation

ECH 19

Page 20: Internal Medicine Topics

-hypothalamus secretes thyrotropin-releasing hormone (TRH; tripeptide) stimulates production of thyroid stimulating hormone (TSH; glycoprotein) in anterior pituitary TSH production and release of thyroxine (T4) and triiodothyronine (T3) from thyroid gland negative FB on TSH production-T4 is 5-fold less active than T3, 85% of T3 is formed from peripheral conversion of T4-most T3 & T4 in plasma is protein bound, mainly to thyroxine-binding globulin (TBG); the unbound portion is the active part-the thyroid hormones cell metabolism via nuclear receptors, and are vital for growth and mental development. They alsocatecholamine effect

-the following are indications for screening of thyroid function: AF, hyperlipidemia, DM, amiodarone or lithium medication, Down’s syndrome, Turner’s syndrome, Addison’s disease

Thyroid function tests (TFTs)-basic tests: measurement of free T4 & T3 is more useful than total levels as the latter are affected by TBG. Total T4 & T3 is when TBG is (and vice versa). Free T3 & T4 levels are unaffected

-TBG is in pregnancy, estrogen therapy (HRT, oral contraceptive pill) and hepatitis

-TBG is in nephritic syndrome and malnutrition (protein loss), drugs (androgens, corticosteroids, phenytoin), chronic liver disease and acromegalyTSH, T4 HypothyroidismTSH, normal T4 Treated hypothyroidism or subclinical

hypothyroidismTSH, T4 TSH secreting tumor or thyroid hormone

resistanceTSH, T4 orT3 HyperthyroidismTSH, normal T4 & T3 Subclinical hyperthyroidismTSH, T4 and T3 Sick euthyroidism or pituitary diseaseNormal TSH, abnormal T4 Consider changes in thyroid-binding globulin,

assay interference, amiodarone (antiarrhythmic agent) or pituitary TSH tumor

-if hypothyroidism is suspected or for monitoring replacement therapy: ask for T4 and TSH. T3 provides no extra information-sick euthyroidism: in any systemic illness, TFTs may become deranged (all values are low). Repeat test after recovery-assau interference: caused by Ab.’s in the serum

Other tests:-thyroid autoantibodies: antithyroid peroxidase (TPO) Ab or antithyroglobulin Ab may be increased in AI thyroid disease (Hashimoto or Graves’) – if positive in Graves’, there is risk of developing hypothyroidism at a later stage-TSH receptor Ab: may be in Graves’ d. (useful in pregnancy)-Serum thyroglobulin: monitoring treatment of cc., and in detection of self-medicated hyperthyroidism (low value)-US: distinguish cystic from solid nodules (if there is a solitary large nodule, or dominant nodule in multinodular goiter, FNA is indicated to look for thyroid cc.)

ECH 20

Page 21: Internal Medicine Topics

-isotope scan: 123Iodine or 99Technetium pertechnetate. Useful for determining the cause of hyperthyroidism, for detection of retrosternal goiter, ectopic thyroid tissue or thyroid metastases areas show increased (hot), decreased (cold), or the same (neutral) uptake of isotope as the remaining thyroid; 20% of cold nodules are malignant

Hyperthyroidism

Thyrotoxicosis: clinical and biochemical effect of excess thyroid hormone-symptoms: weight loss despite appetite (paradoxical weight gain in 10-30%), heat intolerance, sweating, diarrhea, tremor, irritability, emotional lability, itch, psychosis, oligomenorrhea

-signs: tachycardia AF, warm peripheries and palmar erythema, hair thinning, lid lag (von Graefe’s sign), lid retraction (stare, Dalrymple sign). Goiter or thyroid nodules may be present

-Graves’ only: eye disease (exophthalmos, ophthalmoplegia), pretibial myxedema, thyroid acropachy (clubbing, painful finger and toe swelling, periosteal reaction in limb bones)

-eye disease occurs in 25-50%, main risk factor is smoking. Eye disease may be the first presenting sign of Graves’ d., the pt can be euthyroid, hypothyroid or hyperthyroid. Retro-orbital inflammation and lymphocyte infiltration results in swelling of the orbit. Clinical diagnosis/CT&MRI of orbits may reveal enlarged eye muscles-tests: TSH (neg. FB), T4 and T3

-there may be mild normocytic anemia, mild leukopenia, ESR, Ca2+, LFT (liver function test)

-also check autoAb, isotope scan if the cause is unclear, to detect nodular d. or subacute thyroiditis (test visual fields, acuity, eye movements if eye disease is present)

Causes-Graves’ d. (Basedow’s d. in Hungary): ♀:♂ ≈ 9:1, 30-50YO; AI disease with stimulatory TSH receptor Ab’s (thyroid-stimulating immunoglobulin, TSI; thyroid growth immunoglobulin, TGI; antimicrosomal etc. Ab’s; also react with orbital autoAg’s) diffuse goiter. Associated with other AI d.’s: vitiligo (depigmentation in patches of skin), type 1 DM, Addison’s. Incidence in HLA-DR3 & HLA-B8-positive individuals-Toxic multinodular goiter (Plummer disease): in elderly and iodine-deficient areas; nodules secrete thyroid hormones

-surgery is indicated if there are compressive symptoms (dysphagia, dyspnea)-Toxic adenoma: solitary nodule (hot on isotope scan) producing T3 & T4. The rest of the gland is suppressed. Treatment: radioiodine-Hyperthyroxinemia can also occur because of thyroiditis (Hashimoto’s, de Quervain’s,; usually progress to hypothyroidism), oral consumption of excess thyroid hormone tablets, hamburger hyperthyroidism (ground beef contaminated with thyroid tissue), amiodarone medication, postpartum

ECH 21

Page 22: Internal Medicine Topics

thyroiditis (usually self-limited), pituitary hyperfunction (TSH secondary hyperthyroidism), struma ovarii (ovarian teratoma made up of thyroid tissue)

Treatment--blockers for rapid control of symptoms, anti-thyroid medication: titration of carbimazole (SE: agranulocytosis), carbimazole + thyroxine simultaneously (less risk of iatrogenic hypothyroidism)-radioiodine 131I: most become hypothyroid post-treatment. CI: pregnancy, lactation-thyroidectomy: risk of damage to recurrent laryngeal nerve (hoarse voice) and hypoparathyroidism

-complications: HF (thyrotoxic cardiomyopathy), angina, AF (10-25%), osteoporosis, ophthalmopathy, gynecomastia

-thyrotoxic storm/hyperthyroid crisis: may occur when a thyrotoxic patient becomes very sick or physically stressed. Symptoms: fever, agitation, confusion, coma, tachycardia, AF, goiter, thyroid bruit, diarrhea & vomiting, ‘acute abdomen’ picture

-subclinical hyperthyroidism: TSH, normal T4/T3; ongoing trials are assessing whether treatment prevents subsequent complications (AF, osteoporosis)

Hypothyroidism

-symptoms: tiredness, lethargy, depression, cold intolerance, weight gain, constipation, menorrhagia, hoarse voice, poor cognition/dementia, myalgia, bradycardia, dry skin and hair, non-pitting edema (eyelids, hands, feet), cerebellar ataxia (lack of coordination of muscle movements), slow relaxing reflexes, peripheral neuropathy, toad-like face. Goiter may be present, or signs of congestive cardiac failure or pericardial effusion-Dx: TSH, T4 (in secondary hypothyroidism T4 and /TSH due to lack from the pituitary), cholesterol and TAG may be . Occasionally normochromic macrocytic anemia-clinical syndromes:-myxedema in adults: cutaneous and dermal edema secondary to increased deposition of connective tissues (glycosaminoglycans, hyaluronic acid, other mucopolysacccharides)

-cretinism in children: due to iodine def., enzyme def., maldevelopment of thyroid (failure of fetal thyroid to descend), transplacental transfer of Ab’s. Cause severe mental retardation, retarded bone development and dwarfism, macroglossia, protuberant abdomen

-primary hypothyroidism-Riedel’s thyroiditis: unknown etiology (probably AI); glandular atrophy

and thyroid replacement by fibrous tissueAutoimmune: associated with type 1 DM; Addison’s d., pernicious anemia

ECH 22

Page 23: Internal Medicine Topics

-primary atrophic hypothyroidism: ♀:♂ ≈ 6:1; common; diffuse lymphocytic infiltration (no goiter)

-Hashimoto’s thyroiditis: as above, plus goiter due to lymphocytic and plasma cell infiltration. May be hypothyroid/euthyroid – occasionally initial period of hyperthyroidism (Hashitoxicosis). High autoAb titer

Acquired-iodine deficiency: commonest cause world-wide-post-thyroidectomy or radioiodine treatment-drug-induced: antithyroid drugs, amiodarone, lithium, iodine-subacute (de Quervain, granulomatous) thyroiditis: painful; temporary

hypothyroidism after hyperthyroid phase-secondary hypothyroidism from hypopituitarism is rare

-Associated with Turner’s, Down’s, CF, primary biliary cirrhosis, POEMs syndrome (polyneuropathy, organomegaly, endocrinopathy/edema, m-protein band from a plasmacytoma + skin pigmentation/tethering). Genetic: dyshormonogenesis: genetic (mostly AR) defect in hormone synthesis (Pendred’s syndrome: with deafness; uptake on isotope scan)

-treatment: levothyroxine (T4)

!Myxedema coma:-Signs & symptoms: looks hypothyroid; >65yrs; hypothermia, hyporeflexia, glucose, bradycardia, coma, seizures-History & precipitants: prior surgery or radioiodine for hyperthyroidism; infection, MI, stroke, trauma-Examination: goiter, cyanosis, HF-Take blood samples, treat symptoms (O2, T3, hydrocortisone, saline, AB, etc.)

-subclinical hypothyroidism: TSH, normal T4/T3; no obvious symptoms. Common: about 10% of ppl >55yrs, risk increases with TSH, presence of autoAb’s, male sex; monitor TSH (treat with thyroxine in pt’s with TSH>10)

!amiodarone can cause both hyper- and hypothyroidism; effects are due to drug itself + iodine contained within it. Hypothyroidism is caused by iodine excess, inhibiting hormone synthesis and release

Thyroiditis

Hashimoto’s thyroiditis-women 60-70yrs-AI; common cause of hypothyroidism; may have an early transient hyperthyroid phase (Hashitoxicosis)-slow course, modestly enlarged and nontender thyroid. Goiter is due to lymphocytic and plasma cell infiltration. Pt is often euthyroid at first; hypothyroidism develops late when the thyroid is shrunken and scarred

ECH 23

Page 24: Internal Medicine Topics

-histo: lymphocytic infiltrates with germinal center formation; atrophic thyroid follicles; Hürthle cells (epith. cells with eosinophilic granular cytoplasm) are prominent-associated with various autoAb’s (high titer; antithyroglobulin, antithyroid peroxidase, anti-TSH receptor, anti-iodine receptor Ab’s)-incidence is in HLA-DR5- and HLA-B5-positive individuals; Hashimoto’s may also be associated with other AI disorders: pernicious anemia, DM, Sjögren syndrome (xerostomia, keratoconjunctivitis sicca + conn. tissue or AI disease)

Subacute (de Quervain’s, granulomatous) thyroiditis-F>M-painful goiter; focal destruction of thyroid tissue, granulomatous inflammation-viral etiology (mumps, coxsackievirus)-self-limited course of several weeks’ duration consisting of a flu-like illness along with pain and tenderness of the thyroid, sometimes with transient hyperthyroidism

Postpartum thyroiditis (PPT)-usually self-limited, affects about 5% of all women within a year after giving birth-may involve hyper- or hypothyroidism-believed to result from the modifications to the immune system necessary in pregnancy-histo: lymphocytic infiltration

Subacute lymphocytic thyroiditis/silent thyroiditis-similar to PPT; most likely AI etiology-small goiter without tenderness; hyperthyroidism hypothyroidism euthyroidism (resolving thyroiditis)

-Acute infectious thyroiditis: rare; usually bacterial etiology

-Riedel’s thyroiditis: rare, F>M; unknown etiology (probably AI); glandular atrophy and thyroid replacement by fibrous tissue ( hard thyroid gland; adhesion to surrounding structures; most pt’s remain euthyroid, 30% hyperthyroid

Sporadic and endemic goiter

-endemic goiter is goiter occurring with high frequency (>5%) in iodine-deficient geographic areas; sporadic goiter is goiter caused by similar mechanisms in non-iodine-deficient areas-iodine-deficient areas are typically remote inland areas where no marine food is eaten (mountainous regions where food is grown in soil poor in iodine)-endemic goiter is particularly common in Western Pacific, South-East Asia and Africa [India: 500 million def., 54 mill. goiter, 2 mill. cretinism]-iodine deficiency is the most common cause for goiter worldwide

ECH 24

Page 25: Internal Medicine Topics

-iodine is necessary for the synthesis of T4 & T3. When levels of thyroid hormones fall because of iodine deficiency, thyrotropin-releasing hormone (TRH) is produced by the hypothalamus. TRH then stimulates the pituitary gland to make thyroid stimulating hormone (TSH), which stimulates the thyroid gland’s production of T4 and T3. It also causes the thyroid gland to grow in size by increasing cell division-iodine def. hypothyroidism (lack of thyroid hormones) cretinism. Thus, iodine deficiency is a highly preventable cause of mental retardation. Combatted by addition of small amounts of iodine to table salt. A proposal to mandate the use of iodized salt in most commercial breadmaking is expected to be adopted in 2009-treatment: iodine supplementation (if goiter is untreated for >5yrs, iodine suppl./thyroxine treatment may not reduce the size because the gland is permanently damaged)

Benign and malignant tumors of the thyroid

-feel the thyroid for size, shape, tenderness, and mobility. Percuss for retrosternal extension-about 10% of single thyroid lumps are malignant-do T3 & T4; US (see if lump is cystic, complex or part of a group of lumps); radionuclide scans show malignant lesions as hypofunctioning/cold; a hyperfunctioning/hot lesion suggests adenoma); FNA, do cytology on the fluid-no clinical/lab test is good enough to tell for sure if follicular neoplasms found on FNA are benign, so such pt’s are normally referred for surgery

Benign adenomas-often solitary-present clinically as nodules, varied histological pattern (eg. follicular, Hürthle cells)-most often nonfunctional; can occasionally cause hyperthyroidism

Malignant tumors-Papillary carcinoma

-most common thyroid cancer, 60%; often in young-histo: papillary projections into gland-like spaces; tumor cells have

“ground-glass” (“Orphan Annie”) nuclei; calcified spheres (psammoma bodies) may be present

-can be a long-term consequence of prior radiotherapy to the neck-can be associated with changes in chr. 10 (paracentric inversion,

reciprocal translocation bw. 10/17) constitutively active tyrosine kinase domain of ret [fusion genes are referred to as ret-PTC (papillary thyroid cc.) and are analogous to bcr-abl fusion in CML]

-Rx: total thyroidectomy node excision 131I, T4 to suppress TSH-better prognosis than other forms of thyroid cancer (best prognosis if

young and ♀)-Follicular cc.

-25%; middle-aged population; spreads early via blood (bone, lungs)

ECH 25

Page 26: Internal Medicine Topics

-histo: relatively uniform follicles-Rx: total thyroidectomy + T4 suppression + 131I

-Medullary cc.-5%; originates from C cells of the thyroid (parafollicular cells; produce

calcitonin, a calcium-lowering hormone)-histo: sheets of tumor cells in an amyloid-containing stroma-can be associated with MEN IIa and IIb (III) (20%)-Rx: thyroidectomy + node clearance

-Lymphoma-5%; ♀:♂ ≈ 3:1; may present with stridor or dysphagia-Rx: chemoradiotherapy (MALT origin is associated with good prognosis)

-Anaplastic-rare; ♀:♂ ≈ 3:1; elderly population-poor prognosis

Post-op complications-recurrent (superior) laryngeal nerve palsy ( hoarseness) occur permanently in 0.5% and transiently in 1.5%-pt’s voice will be different for a few days post-op because of intubation and local edema-hypoparathyroidism (hypocalcemia) that is permanent in 2.5%-hypothyroidism-thyroid storm-tracheal obstruction due to hematoma in the wound

Hyperparathyroidism

The sole function of the parathyroid glands is to maintain the body's calcium level within a very narrow range, so that the nervous and muscular systems can function properly. They secrete parathyroid hormone (parathormone, PTH) with effects antagonistic to those of calcitonin-When blood ionized calcium levels drop below a certain point, calcium-sensing receptors in the parathyroid gland are activated to release hormone into the blood. Secretion is not under pituitary control-PTH blood calcium levels by

-stimulating osteoclasts to break down bone and release Ca2+ and PO3-4

-GI absorption of calcium by activating vit. D-promoting calcium uptake by the kidneys (and PO3-

4 reabsorption)

Primary hyperparathyroidism-most often caused by parathyroid adenoma (80-85%)(a few cases are caused by primary parathyroid hyperplasia (15-20%) or carcinoma(<0.5%)); less often caused by production of PTH-like hormone by nonparathyroid malignant tumors (bronchiogenic squa. cell cc., renal cell cc.)

-can occur as part of MEN I and MEN IIa-pt’s are often asymptomatic, but may present with weakness, tiredness, depression, dehydration, polyuria, polydipsia, renal stones, abdominal pain,

ECH 26

Page 27: Internal Medicine Topics

pancreatitis, ulcers, bone pain or fractures, osteopenia/osteoporosis. There may be BP (long-term cardiovascular risk and mortality in untreated pt’s)-labs: hypercalcemia, hypercalcuria; Se phosphorus, tubular resorption of phosphorus, urinary phosphorus; Se ALP; Se PTH-clinical characteristics:

-osteitis fibrosa cystica: cystic bone changes due to osteoclastic resorption (von Recklinghausen d. of bone). Fibrous replacement of resorbed tumor may lead to the formation of non-neoplastic tumor-like masses (“brown tumor”)

-metastatic calcifications (nephrocalcinosis)-renal calculi-peptic duodenal ulcer (hypercalcemia predisposes to peptic ulcer)

-Rx: surgical excision of adenoma/all 4 glands

Secondary hyperparathyroidism-compensatory hyperplasia in response to Ca2+ in serum (hypocalcemia of chronic renal disease – vit. D conversion in kidney is impaired intestinal calcium absorption)-labs: hypocalcemia (may be slight); Se phosphorus (due to renal failure); Se ALP; PTH-diffuse osteoclastic bone disease; metastatic calcifications

Tertiary hyperparathyroidism-persistent parathyroid hyperfunction in spite of correction of hypocalcemia and preexisting secondary hyperparathyroidism; Ca2+, PTH-often caused by development of adenoma in previous hyperplastic gland

1-hydroxylase deficiency (vitamin D-dependent rickets)-impaired hydroxylation of 25-hydroxycholecalciferol ( active vit. D) in the kidneys-absorption of calcium, PTH secretion

MEN syndromes: AD, more than one endocrine organ is hyperfunctional. May be associated with hyperplasias or tumors-MEN I (Wermer syndrome): mutations in MEN I gene (tumor suppression gene)

-hyperplasias/tumors of pituitary, parathyroid, pancreatic islets (may include hyperplasias/tumors of thyroid or adrenal cortex, carcinoid tumors)

-pancreatic component may be Zollinger-Ellison syndrome, hyperinsulinism, or Verner-Morrison syndrome/pancreatic cholera-MEN IIa (Sipple syndrome)

-pheochromocytoma, medullary cc. of thyroid, hyperparathyroidism-linked to mutations in ret proto-oncogene (receptor tyrosine kinase;

clinically important – if ret mutation is found in pt with pheochromocytoma, prophylactic thyroidectomy is indicated)[-MEN IIb (III)

-pheochromocytoma, medullary cc., multiple cutaneous neuromas or ganglioneuromas (NO hyperparathyroidism)

-Marfanoid appearance-different mutations in ret proto-oncogene compared with MEN IIa]

ECH 27

Page 28: Internal Medicine Topics

Hypoparathyroidism

-PTH-tests: Ca2+, PO3-

4 (or normal), ALP normal-most common cause is accidental surgical excision during thyroidectomy (secondary hypoparathyroidism). In rare instances, it is associated with congenital thymic hypoplasia (DiGeorge syndrome). Can also be of AI origin or due to hypomagnesemia (secondary; magnesium is required for PTH secretion)-severe hypocalcemia manifest by increased neuromuscular excitability and tetany is characteristic-Rx: alfacalcidol

Pseudohypoparathyroidism-similar to hypoparathyroidism: calcium, phosphate, PTH (ALP /)-multihormone resistance involving PTH and pituitary hormones TSH, LH, FSH-end-organ unresponsiveness of the kidney to PTH-shortened 4th and 5th metacarpals and metatarsals, short stature, other skeletal abnormalities, round face, mental retardation-mutations in GNAS1 – in the kidney and pituitary, there is expression only of the maternally inherited chromosome maternal inheritance of end-organ unresponsiveness. Skeletal abnormalities result from inheritance of mutant GNAS1 from either parent-similar skeletal abnormalities, without hormone dysfunction, characterize pseudopseudohypoparathyroidism, which requires transmission of a mutant paternal GNAS1 allele-Rx: alfacalcidol

AI polyendocrine syndromes (AI disorders cluster into two defined syndromes):-Type 1: AR; rare

-mutations of AIRE (auto immuneregulator) gene, chr. 21-Addison’s d., chr. mucocutaneous candidiasis, hypoparathyroidism-also associated with primary hypogonadism, pernicious anemia, AI

primary hypothyroidism, chr. active hepatitis, vitiligo (depigmentation in patches of skin), alopecia (hair loss)-Type 2: HLA D3 and D4 linked; common

-polygenic cause-Addison’s d., type 1 DM (in 20%), AI thyroid d. – hypothyroidism or

Graves’ d.-also associated with primary hypogonadism, vitiligo, alopecia, pernicious

anemia, chr. atrophic gastritis, coeliac d., dermatitis herpetiformis

Diseases of the breast

Mastitis-acute mastitis: common during lactation (S. aureus inf. most common)

ECH 28

Page 29: Internal Medicine Topics

-fat necrosis: trauma, surgery; may produce palpable mass or lesion on mammography

Fibrocystic disease-the most common cause of a palpable mass in pt.’s bw. 25-50yrs-lumpy breasts (usually bilateral) with midcycle tenderness; most often involves the upper outer quadrant-thought to result from activity or sensitivity to estrogen, or progesterone activity-prolif. changes are associated with an risk of cancer (esp. when hyperplastic epithelium demonstrates atypia

Nonproliferative fibrocystic changes Proliferative fibrocystic changesFibrosisCysts (blue-domed)Apocrine metaplasiaMicrocalcifications

Ductal hyperplasia atypiaSclerosing adenosisSmall duct papillomas

Benign neoplasms-Fibroadenoma: most common benign breast tumor in women <25

-palpable, round, movable, rubbery mass-histo: proliferation of benign stroma, ducts, and lobules

-Phyllodes tumor (cystosarcoma phyllodes): fibroadenoma variant, older pt population (50s)-large tumor with both stroma + glands; malignant in 10-20%

-ulceration of overlying skin; cystic spaces contain projections from the cyst walls and myxoid contents are characteristic-Intraductal papilloma: presents as bloody (serous) nipple discharge

-benign tumor of the major lactiferous ducts (DDx: cc.)-Adenoma of the nipple: serous or bloody discharge; palpable mass. Can be mistaken for malignancy

Malignant neoplasms-carcinoma of the breast is the second most common malignancy of women (after lung cc.)-1% of all breast cancers are found in men-most common cause of a breast mass in postmenopausal women-most often involves the upper outer quadrant; frequent sites of metastasis include axillary LNs, lung, liver, and bone-presence of estrogen and progesterone receptors in some tumors is correlated with better prognosis and is thought to be a predictor of efficacy of antiestrogen therapy. Other prognostic indicators: type and size of tumor, extent of LN involvement, DNA ploidy. Hyperexpression of c-erbB2 (HER-2/neu) is associated with a poorer prognosis-examination: inspect (arms up and down). Note position, size, consistency, mobility, fixity, local lymphadenopathy. Any nipple discharge/inversion? Is the skin involved: dimpling; ulceration; peau d’orange?

-clinical examination + histology/cytology + mammography + US-predisposing factors:

-age (incidence with age)

ECH 29

Page 30: Internal Medicine Topics

-first-degree relative with breast cancer-5-10% are hereditary (p53 germ-line mut. – Li Fraumeni syndrome;

BRCA-1 – chr. 17q21, also ass. with ovarian malignancies; BRCA-2 – chr. 13q12-13)

-history of breast cancer in one breast-early menarche, late menopause-obesity-nulliparity-first pregnancy after 30yrs of age-high animal fat diet-proliferative fibrocystic disease with atypical epith. hyperplasia(-exogenous estrogens (postmenopausal estrogen therapy))

-treatment: mastectomy or lumpectomy with radiation; axillary dissection (local disease); tamoxifen, chemotherapy (metastatic disease)-Preinvasive lesions

-ductal cc. in situ (DCIS): comedocarcinoma; tumor cells fill ducts; tumor cell necrosis results in a cheese-like consistency

-lobular cc. in situ (LCIS): clusters of neoplastic cells fill intralobular ductules and acini; may to invasive cc. in the same or contralateral breast; often bilateral at the time of initial diagnosis

-Paget disease of the nipple: eczematoid lesion of nipple/areola; Paget cells: large cells surrounded by a clear halo-like area; intraepidermal invasion; underlying ductal cc. is almost always present

-Invasive ductal cc. (scirrhous cc.): most common (>80%), tumor cells arranged in cords, islands and stroma embedded in a dense fibrous stroma (firm consistency)

-Invasive lobular cc.: often multicentric or bilateral; cells arranged in a linear fashion (“Indian-file” appearance); better prognosis than that for invasive ductal cc.

-Mucinous (colloid) cc.: pools of extracellular mucus surrounding clusters of tumor cells; gelatinous consistency; better prognosis

-Tubular cc.: rarely metastasizes; excellent prognosis

-Medullary cc.: cellular with scant stroma; soft, fleshy consistency; lymphocytic infiltrate; better prognosis

-Inflammatory cc.: lymphatic involvement of skin by underlying cc. red, warm, edematous skin (peau d’orange: thickened skin resembles orange peel); poor prognosis

The carcinoid syndrome

-synonyms: Thorson-Bioerck sy., Argentaffinoma sy., Cassidy-Scholte sy., Flush sy.

ECH 30

Page 31: Internal Medicine Topics

-a carcinoid tumor is a small-growing, malignant endocrine tumor. It is also called an apudoma as it arises from APUD (enterochromaffin) cells from structures such as Vater’s ampulla, pancreas, prostate (neural crest cell origin; 2/3 of carcinoids are found in the GI tract)

-most sources (eg. Robbins, wiki, OHCM) states appendix as most common site (others claim that the foregut is the most involved site)

-rectal and appendiceal carcinoids rarely metastasize-tumor localization may be difficult: do CXR + chest/pelvis CT/MRI.

Barium swallow and follow-up examination of the intestine may occasionally show the tumor. Capsule video endoscopy can also be used. Laparotomy is the definitive way to localize the tumor-carcinoid tumors account for 75% of GI endocrine tumors-10% are part of MEN I, 10% occur with other neuroendocrine tumors

-the carcinoid syndrome results from metastasis of carcinoid tumors – effects and symptoms are usually due to production of serotonin (5-HT/5-hydroxytryptamine; vasoactive substance) released into systemic circulation escaping hepatic degradation. Carcinoid syndrome occurs in about 1% of all pt.’s with carcinoids and 20 % of those with widespread metastasis (according to OHCM: 5%, implies hepatic involvement)-solid, yellow-tan appearance on transection-histo: neoplastic cells may form discrete islands, trabeculae, strands, glands, or undifferentiated sheets. Pink granular cytoplasm; minimal variation in cell and nuclear size; infrequent/absent mitosis. Most carcinoids contain chromogranin A (measurement can be used for Dx), synaptophysin, neuron-specific enolase

-manifestations are thought to occur from elevated levels of 5-HT and its metabolite, 5-IHAA (5-hydroxyindoleacetic acid) blood and urine levels are elevated (>25 mg/day) in most pt.’s with classic symptoms. Contribution of other secretory products (histamine, bradykinin, PGs) has not been excluded-the ‘classic’ symptoms are cutaneous flushing, diarrhea, bronchoconstriction, right-sided cardiac valve disease, cyanosis

-vasomotor disturbances (most pt.’s): cutaneous flushing, apparent cyanosis

-intestinal hypermotility (most pt.’s): diarrhea, cramps, nausea, vomiting-asthmatic bronchoconstrictive attacks (1/3 of pt.’s): cough, wheezing,

dyspnea-hepatomegaly: nodular, related to hepatic metastasis-niacin def. (shunting of niacin 5-HT)-systemic fibrosis-cardiac involvement: pulmonary & tricuspid valve thickening/stenosis,

endocarditis and endocardial fibrosis of right ventricle (bronchial carcinoids affect left side when the lung is incapable of detoxifying the5-HT)

-retroperitoneal & pelvic fibrosis-collagenous pleural and intimal aortic plaques

-treatment: symptomatic relief; somatostatin analogue (octreotide; neutralizes 5-HT, urinary 5-HIAA), antiserotonin agent (SE: retroperitoneal fibrosis), antihistamine drug; surgical resection and chemotherapy for qualifying candidates

ECH 31

Page 32: Internal Medicine Topics

-5-year survival rate 90%, widespread disease usually causes death

Principles of alimentation and hyperalimentation

-alimentation consists of intake of food, digestion to extract energy and nutrients, and expulsion of the remaining waste. The major functions of the GI tract is thus ingestion, digestion, absorption, and defecation-normal alimentation requires a proper functioning GI tract (ie. normal anatomy, normal function of secretory organs, absorptive areas etc.)-the time taken for food or other ingested objects to transit through the GI tract varies depending on many factors, but roughly, it takes 2,5 to 3 hours after meal for 50% of stomach contents to empty into the intestines. Total emptying of the stomach takes 4 to 5 hours. Subsequently, 50% emptying of the small intestine takes 2.5 to 3 hours. Finally, transit through the colon takes 30 to 40 hours

-essential nutrients: -FA: linolenic acid, linoleic acid-AAs: his, ile, lys, leu, met, phe, thr, trp, val (arg in children only)-vitamins: B1, 2, 3, 5, 6, 7, 9, 12; C, D, E, K, folic acid-minerals

-protein turnover ≈ 300-400 g/day; in most cases a diet in which 12% of the energy is supplied as protein is adequate

-Recommended Dietary Allowance (RDA) (40-50yrs)Substance Amount (males) Amount (females)Water (from food, beverages, drinking water)

3.7 L/day 2.7 L/day

Carbohydrates 130 g/day 130 g/dayProtein (0,8 g/kg BW) 56 g/day 46 g/dayFiber 38 g/day 25 g/dayFat 20-35% of caloriesLinoleic acid 17 g/day 12 g/dayLinolenic acid 1.6 g/day 1.1 g/dayCholesterol As low as possibleTrans fatty acids As low as possibleSaturated fatty acids As low as possibleAdded sugar No more than 25% of calories

-carbohydrates: digestion starts in the mouth (salivary -amylase), continues in the intestines (pancreatic -amylase) pancreatic dextrinase turns dextrin maltose. Disaccharides are broken down to monosaccharides by enzymes attached to the epith. cells of SI. Absorption of monosaccharides occur in the jejunum (glc & gal –cotransported with Na+ by SGLT; frc passively diffuse via GLUT5) Monosaccharides leave the intestinal cells to enter hepatic circulation by diffusion through GLUT2

-lipids:

ECH 32

Page 33: Internal Medicine Topics

-TAG digestion: lingual lipase, gastric lipase, pancreatic lipase slowly hydrolyse TAG 1,2-DAG + FA; 1,2-DAG 2-MAG + FA. Bile acids emulsify lipid droplets (surface area) and activate pancreatic lipase

-PL digestion: phospholipases (mainly pancreatic) hydrolyze PLs glycerol, Pi, FA, X

-cholesterol ester digestion: hydrolyzed by pancreatic cholesterol esterase

-absorption: primary products of lipid degradation in jejunum is FFA, free cholesterol, 2-MAG form mixed micelles absorbed by brush border membrane of intestinal mucosal cells. SCFA & MCFA may be released into portal circulation carried by albumin. Other products are resynthesized to cholesterol ester, phospholipids, triglycerides packed in chylomicrons (lipoproteins) and released into lymphatic circulation thoracic duct left subclavian vein tissues

-proteins: digestion starts in the stomach and is done by HCl (denaturation) and pepsin (peptide bond cleavage) duodenal cells detect protein fragments and AAs secrete CCK stim. pancreas to secrete zymogens (trypsinogen, chymotrypsinogen, proelastase, procarboxypeptidase) & secretin trypsinogen is converted to trypsin by enteropeptidase, subsequently activates the other zymogens the serine proteases/endopeptidases (trypsin, chymotrypsin, elastase, carboxypeptidase) + aminopeptidase (from intest. epith. cells) cut additional peptide bonds free AAs and dipeptides. Absorption occurs in the jejunum, where free AAs are taken up by epith. cells via specific active Na+ dependent cotransporter. Dipeptides and tripeptides are transported via non-Na+-dependent molecules and hydrolyzed by cytosolic peptidases before being released into portal system

-free AAs are transported in blood enter cells against their gradient through ATP-dependent transporters

-vitamins:-water-soluble vitamins (B complex vitamins, B1 (thiamine), B2 (riboflavin), B3 (niacin), B6

(pyridoxine), and B12 (cobalamine); folic acid; vit. C (ascorbic acid)) are not stored in the body; regular uptake is essential (exception – vit. B12 is stored in the liver, takes months-yrs for def. to develop). Found in whole grain cereals, green leafy vegetables, fish, meat, dairy foods (B vit.’s except B12, folic acid), foods of animal origin (vit. B12), fruits, vegetables, various meats and milk (vit. C). Are readily excreted from the body, to the degree that urinary output is a strong predictor of vitamin consumption

-fat-soluble vitamins (A, D, E, K, found in a variety of foods, esp. fruits and vegetables) are absorbed with the help of lipids

Hyperalimentation refers to a state where quantities consumed are greater than appropriate: the term can also be used to describe ingestion to compensate for past deficiencies (can also occur in parenteral nutrition)-hyperalimentation can cause an osmotic diuresis due to an increased load of urea from protein catabolism. Also associated with opportunistic infections by C. albicans

ECH 33

Page 34: Internal Medicine Topics

-can result in overweight/obesity, vitamin poisoning, or mineral overload (toxicity)

- Pica is a medical disorder characterized by an appetite for substances largely non-nutritive (e.g., clay, coal, soil (geophagy), feces (coprophagia), chalk, glass (hyalophagia),paper, soap, mucus, ash, gum etc.) or an abnormal appetite for some things that may be considered foods, such as food ingredients (e.g., flour, raw potato, raw rice, starch (amylophagia), ice cubes (pagophagia), salt, blood)

-seen particularly in pregnant women, small children (often with developmental disabilities), and patients with iron deficiency anemia

The disorders of lipid metabolism

-raised or abnormal levels of lipids and/or lipoproteins in blood-fatty oxidation disorders or lipid storage disorders inborn errors of metabolism, enzyme defects affect the ability to oxidize FAs in order to produce energy within muscles, liver etc.-dyslipidemia: Ch, TAG, LDL, HDL (often due to visceral adiposity)-INS R + visceral adiposity (waist circumference>102/88 cm ♂/♀) + TAG + HDL-Ch + HTN = metabolic syndrome (>3 signs)-normal lab values

-serum Ch 3.6-5.2 mM-serum TAG 0.8-2.3 mM-HDL Ch 1-1.6 mM (higher in ♀)-LDL Ck <4.1 mM (preferably <3.4 mM)

-serum lipid determination is important for Dx of atherosclerosis; cholesterol is a major risk factor for coronary heart disease (CHD)

-modified (oxidated) LDL cannot bind to Rs on hepatocytes blood level . They bind to scavenger Rs on MPHs foam cells (lipid accumulation)

-inflammation ROS LDLox-DM glc LDLgly

-screening should be performed in ppl with CHD or risk (DM, BP), family history of hyperlipidemia, xanthoma or xanthelasma (cholesterol depositions under skin), corneal arcus before 50yrs old-elevated serum levels may be due to inherited or acquired reasons: sedentary lifestyle; high fat, high carbohydrate diet-laboratory diagnosis: lipoprint, electrophoresis (from blood, urine)-symptoms: xanthelasma palpebrarum, xanthomas (striata palmaris, tuberosum ++)-risk factors: male sex, high total Ch, high LDL-Ch, low HDL-Ch, HTN, DM/IGT (‘new’, assumed risk factors: homocysteine, CRP, fibrinogen, t-PA)-causes of secondary hyperlipoproteinemias (acquired): obesity, DM, hypothyroidism, renal disease (nephrosis, nephritic syndrome, end-stage renal disease), cholestasis (cholelithiasis, cholecystitis, cancer of gall bladder – malabsorption of lipids due to bile acids – additional symptom: steatorrhea), alcoholism, estrogen treatment (competitive degradation in the liver), liver

ECH 34

Page 35: Internal Medicine Topics

disease (cirrhosis,) GSDs, Crohn’s disease (and other malabsorption syndromes leading to fat absorption)

-congenital enzyme defects:-CoA dehydrogenase deficiencies

-carnitine:palmitoyltransferase deficiencies-lipid storage disorders: AR or XR inheritance; amount of lipid

accumulate in cells and tissues; leads to permanent cellular and tissue damage over time (esp. in brain, PNS, liver, spleen, bone marrow). These disorders are part of the lysosomal storage diseases: GM1 & GM2 gangliosidoses, (GM2 = Tay-Sachs d.) Gaucher d., Niemann-pick d., Fabry d., fucosidosis, Schindler d., metachromatic leukodystrophy, Krabbe d., multiple sulfatase def., Farber d., Wolman d.

-inherited hyperlipidemiasFredrickson /synonyms Genetic

defectlipoprotein Serum

lipidELFO 24h test at 4C*

I (rare) Buerger-Gruetz sy., primary hyperlipoproteinemia, familial hyperchylomicronemia

LPL or altered apoC2

CHY TAG chy supernatant

IIa Familial hypercholesterolemia

LDL-R def. LDL Ch clear

IIb Combined hyperlipidemia

LDL-R, apoB

LDL & VLDL Ch + TAG

, pre- turbulent

III (rare) Familial dysbetalipoproteinemia

Defect in apoE2 synthesis

IDL (CHY) Ch + TAG

, pre- turbulent

IV Familial hyperlipemia VLDL prod., elimination

VLDL TAG + Ch

pre- opaque

V (rare) Endogenous hypertriglyceridemia

VLDL prod., LPL

VLDL & CHY TAG + Ch

pre-, chy opaque + foamy supernatant

*TAG-containing lipids cause opacity, supernatant is formed in the presence of CHY

-treatment: diet control (I), bile acid sequestrants, statins, niacin, and fibrate-bile acid sequestrants prevent reabsorption of bile acids in the gut

(enterohepatic circulation) bile acid + drug is excreted (hypolipidemic agent)-statins lower cholesterol by inhibiting HMG-CoA reductase Ch

synthesis, LDL-R synth. (clearance of LDL from blood)-niacin reduce total Ch, TAG, VLDL, LDL and increase HDL levels by

blocking the breakdown of fat in adipose tissue-fibrates HDL and TAG (LDL, less effective) by activating PPAR- Rs

(-oxidation, TAG secretion, LPL activity, HDL)

ECH 35

Page 36: Internal Medicine Topics

Protein-calorie undernutrition

-protein def. leads to reduced ability to fight infections (T-ly, phagocytes, Ig, IFN)

Kwashiorkor syndrome-inadequate protein intake with adequate energy intake-usually affects children >1yr who are no longer breast-fed and receive a starch-rich, protein-poor diet-symptoms: severe edema (protein def., oncotic pressure), anemia, malabsorption due to SI villi atrophy, depigmented bands with pale streaking in the hair or skin, thinning hair, diarrhea, dermatitis, growth retardation, fatty liver

Marasmus syndrome-inadequate intake of protein and calories; often coexists with vitamin deficiencies-typically occurs in children <1yr who are not breast-fed and do not have an adequate intake of substitute nutrients-symptoms: thin, small relative to age, absence of body fat, muscle atrophy, retarded growth, mental retardation, INS-better prognosis than in Kwashiorkor (according to wiki)

-it is important to treat not only the symptoms but also the complications of the disorder, including infections, dehydration and circulation disorders, which are frequently lethal and lead to high mortality if ignored

Anorexia nervosa & bulimia nervosa

-anorexia nervosa is self-induced starvation, resulting in marked weight loss; bulimia is a condition in which the pt. binges on food and then induces vomiting-bulimia is more common than anorexia nervosa, and generally has a better prognosis-epidemiology: 1-2% of women, 0.1% of men, average onset 20yrs

-clinical findings of anorexia nervosa are similar to those in severe protein-energy malnutrition. In addition, endocrine effects are prominent

-amenorrhea (due to GnRH LH & FSH; so common it’s a diagnostic feature)

-cold intolerance, bradycardia, constipation, skin and hair changes (due to thyroid hormones)

-dehydration and electrolyte disturbances are common-dry, scaly skin; may become yellow because of excess carotene-decreased bone density (most likely due to low estrogen levels)-anemia, lymphopenia, hypoalbuminemia may be present

ECH 36

Page 37: Internal Medicine Topics

-patients with anorexia nervosa have an increased susceptibility to cardiac arrhythmia and sudden death (hypokalemia)

-bulimic patients binge on huge amounts of food, principally carbohydrates, only to be followed by induced vomiting; weight and gonadotropin levels are maintained near normal-menstrual irregularities are common-major complaints are due to continual use of laxatives and diuretics:

-electrolyte imbalances (hypokalemia predispose to arrhythmias)-pulmonary aspiration of gastric contents-esophageal and stomach rupture

-Dx relies on a comprehensive psychologic assessment of the pt.

Obesity

-because obesity predisposes to several diseases, it is important to define and recognize, to understand its causes, and to be able to initiate appropriate measures to prevent or treat it

-central/visceral obesity is associated with a much higher risk for diseases than is excess accumulation of fat diffusely in subcutaneous tissue-measurements: BMI =(weight in kilograms)/(height in meters)2; skinfold measurements; various body circumferences

BMI State<18.5 Underweight18.5-25 Target25-30 Overweight30-40 Obesity>40 Extreme/morbid obesity

-etiology of obesity involves genetic, environmental, and psychological factors; the majority of cases are not due to metabolic disturbances or genetic disorders-conditions associated with obesity include: genetic (Prader-Willi syndrome, Lawrence-Moon syndrome), hypothyroidism, Cushing’s syndrome and hypothalamic damage (eg. tumor or trauma damage to satiety regions)-normally, the balance bw. energy intake and expenditure is maintained by neural and hormonal mechanisms. The internal set point (“lipostat”) senses the quantity of energy stores (adipose tissue) and regulates food intake as well as energy expenditure

-in recent years, several “obesity genes” have been identified: a key player in energy homeostasis is the LEP gene and its product, leptin (cytokine secreted by adipocytes). The net effect of leptin is to reduce food intake and enhance the expenditure of energy (it is thus suggested that some obese individuals have leptin levels or leptin sensitivity). Neuropeptide Y is a pro-obesity polypeptide secreted by the hypothalamus in response to leptin def.-other neurohormonal mechanisms of regulating energy balance include INS, ghrelin (from stomach; stimulates appetite, may function as a “meal-initiating signal”), and peptide YY (from ileum and colon; satiety signal)

ECH 37

Page 38: Internal Medicine Topics

-obesity is associated with increased risk for INS resistance (release of non-esterified FAs post-receptor defects in INS action), type 2 DM, HTN, hypertriglyceridemia risk of CHD; it is also associated with nonalcoholic steatohepatitis, cholelithiasis, hypoventilation syndrome (pickwickian syndrome), hypersomnolence (due to sleep apnea), osteoarthritis (wear and tear on joints due to BW), ischemic stroke. The theory of relationship between obesity and cancer (esp. endometrial and breast cancers, bowel cancer) is controversial-obesity, dyslipidemia, HTN, and INS resistance are components of metabolic syndrome (syndrome x), which is becoming more and more common, and predisposes to DM type 2 and cardiovascular disease-medication surgery may be considered if the pt. fulfils strict criteria; orlistat lowers fat absorption (SE oily fecal incontinence), sibutramine increases post-ingestive satiety (SE HTN, tachycardia)

Alcoholism

-alcoholism is an important cause of death and disability from several causes ranging from car accidents to homicides-alcohol act on brain cells as a toxin. It changes the fluidity of cell membranes, affecting the transmission of neural impulses, thus causing depression of neural activity. Alcohol has a sedative effect on most neural centers, but the suppression of some centers may lead to a loss of inhibition of others. This may induce a feeling of relaxation or good mood, as well as increased motor activity or uncontrolled emotions. Effects are dose-dependent and can be predicted roughly from the blood alcohol concentration:

-11 mM – sedation-11-33 mM – loss of motor coordination-33-43 mM – delirium-65-87 mM – unconsciousness and coma or resp. arrest

-criteria for the diagnosis of alcohol dependence (>3=alcohol dependence)1. the individual drinks more than he/she means to, often2. the individual is unsuccessful at cutting down3. much time is spent thinking about getting a drink, or when the next

drink will be taken4. there are frequent ill effects from drinking, such as absence from work,

or being drunk or hung over at work5. the individual gives up important nondrinking activities6. the individual continues to drink even though it causes problems in

family or with health7. a tolerance for alcohol has developed8. the individual has physical withdrawal symptoms9. the individual takes substances to relieve withdrawal symptoms

-common pathologic findings in chr. alcoholism include:-fatty liver change, alcoholic hepatitis and (micronodular) cirrhosis-HCC-acute and chr. pancreatitis

ECH 38

Page 39: Internal Medicine Topics

-gastritis-Mallory-Weiss syndrome-other GI signs: diarrhea, peptic ulcers, varices-oral, pharyngeal, laryngeal, esophageal, and gastric cc.-alcoholic (dilated) cardiomyopathy-arrhythmia, BP-anemia (from BM depression, GI bleeding, alcoholism-associated folate

def. megaloblastic anemia, hemolysis, sideroblastic anemia)-aspiration pneumonia-myopathy-peripheral neuropathy-cerebral dysfunction: thiamine deficiency-mediated Wernicke-Korsakoff

syndrome, often associated with hemorrhagic necrosis of mamillary bodies. Manifestations include ataxia, confusion, ophthalmoplegia, nystagmus (Wernicke), memory loss, confabulation (Korsakoff)

-Marchiafava-Bignami syndrome: alcohol-induced corpus callosum necrosis left-handed deficit or constructional ability and several other neurological symptoms

-fetal alcohol syndrome: microcephaly, mental retardation, facial and cardiac defects-acute alcohol intoxication can lead to coma and respiratory arrest, as ethyl alcohol functions as a CNS depressant-alcohol induces P450 enzymes, resulting in increased metabolism of other drugs-methyl alcohol (methanol) is present in solvents, paint remover, and other household chemicals. Abuse by alcoholics (or unintentional intoxication) can lead to necrosis of retinal ganglion cells blindness and inebriation, coma and death, as the metabolites of methanol are toxic for humans. Rx: ethanol (competitive metabolism)-withdrawal symptoms starts 10-72h after last drink; pulse, BP, tremor, confusion, fits, hallucinations-medications exist for managing withdrawal symptoms, as well as for preventing relapse

DDx alcoholic and other liver diseaseASAT ALAT ASAT:ALAT MCV

Alcoholic liver disease

>2

Hepatitis C or <1* Non-alcoholic fatty liver disease

<1 or

Disorders of purine metabolism. Gout-uric acid is the end product of degradation of adenine and guanine-the body contains approx. 1800 mg of uric acid, one third of which is turned over daily-purines that result from normal turnover of cellular nucleic acids, or that are obtained from diet and not

ECH 39

Page 40: Internal Medicine Topics

degraded, can be reconverted to nucleoside monophosphates and used by the body (salvage reactions)-only 2-3% of adults develop gout, even though 10% of the total adult population has hyperuricemia (>0.41 mM)-hyperuricemia can result from overproduction or underexcretion (more common) of uric acid (alcohol, a common trigger of gouty arthritis, can stimulate overproduction and prevent excretion)

Hyperuricemia (gout)-XR-linked inheritance; too much urate is produced-prevalence: 1%, ♂:♀ ≈ 5:1 (it says 9:1 in Damjanov’s pathophys)-symptoms include monosodium urate deposits (kidney) stones, urate crystals (tophus, pl. tophi) in soft tissues, bones, cartilage, and joints inflammation (arthritis), pain, may ulcerate

-when acute, there is severe joint redness, pain, and swelling. Often affects one joint (metatarsophalangeal joint of the big toe, podagra), but may be polyarticular-deposits are formed because urate has low solubility (esp. at acidic pH and low temperatures)-Dx: synovial fluid aspiration of acutely inflamed joint crystals show negative birefringence under polarized light; findings of urate crystals in tissue-origins:

-PRPP overproduction (rare; von Gierke)-absence of purine salvage reactions (eg. HPRT def.)-disturbed ATP metabolism (frc intolerance)-secondary reasons (dietary purine, tissue/cell damage, cytotoxic drugs

used to treat cancer)-alcohol excess (esp. beer),(thiazide) diuretics, leukemia, renal

impairment (reciprocal relationship)-complications: prolonged hyperuricemia may affect the function of renal tubules (urate nephropathy). Deposits in the medulla may cause tissue damage and predispose the pt. to secondary infection (pyelonephritis). Uric acid stones are formed in acid urine, and the incidence of calcium phosphate stones is also increased-treatment: allopurinol (similar to hypoxanthine; inhibit xanthine oxidase accumulation of hypoxanthine and xanthine, which are more soluble in urine excretion). Also include allopurinol when starting chemotherapy to prevent a sharp rise in urate; good hydration and alkalinization of the urine;nucleotides in diet

-in acute gout, a strong NSAID can be used-DDx: calcium pyrophosphate dihydrate (CPPD) arthropathy; similar to gout (pseudogout), presenting as monoarthritis (knee, wrist, hip). Usually spontaneous and self-limiting. Crystals are weakly positively birefringent under polarized light. Rx: analgesia, NSAIDs (steroids)

Lesch-Nyhan syndrome-XR; almost complete deficiency of HPRT (hypoxanthine-guanine phosphoribosyl transferase) no hypoxanthine or guanine salvaging excessive amounts of uric acid

ECH 40

Page 41: Internal Medicine Topics

-pt.’s with Lesch-Nyhan tend to produce kidney stones, exhibit self-mutilation, mental retardation, aggression and involuntary movements-GTP levels of basal ggl. are low, causes the body to poorly utilize vit. B12

-due to lack of salvage: PRPP, IMP, GMP Gln:PRPP amidotransferase (committed step in purine synthesis) has excess substrate and decreased inhibitor available de novo purine synthesis (purine reutilization & purine synthesis results in production of large amounts of urate)

Amenorrhea

-absence of a menstrual period in a woman of reproductive age. Physiological states of amenorrhea are seen during pregnancy and lactation-primary amenorrhea (menstruation cycles never starting) may be caused by developmental problems such as the congenital absence of the uterus, or failure of the ovary to receive or maintain egg cells. Also, delay in pubertal development will lead to primary amenorrhea. It is defined as an absence of secondary sexual characteristics by age 14 with no menarche or normal secondary sexual characteristics but no menarche by 16 years of age-secondary amenorrhea (menstruation cycles ceasing) is often caused by hormonal disturbances from the hypothalamus and the pituitary gland (prolactinoma PRL) or from premature menopause or intrauterine scar formation. Anorexia nervosa will also cause secondary amenorrhea (GnRH LH & FSH). Secondary amenorrhea is defined as the absence of menses for three months in a woman with previously normal menstruation or nine months for women with a history of oligomenorrhea-hypogonadotropic amenorrhea refers to conditions where there are very low levels of serum FSH and LH ( inadequately stimulated ovaries don’t produce enough estrogen to stimulate endometrium); hypergonadotropic amenorrhea refers to conditions with high levels of FSH and LH (ovary or gonad don’t respond to pituitary stimulation). In normogonadotropic amenorrhea, FSH levels are in the normal range (hypothalamic-pituitary-ovarian axis is functional; amenorrhea is due to outflow obstruction etc)-amenorrhea may cause serious pain in the back near the pelvis and spine. This pain can be relieved by a short course of progesterone to trigger menstrual bleeding

Primary amenorrhea-gonadal dysgenesis (eg. Turner Syndrome)-mullerian agenesis (Mayer-von-Rokitansky-Küster-Hauser syndrome (MRKH))-androgen insensitivity syndrome-delay in hypothalamic-pituitary maturation-olfacto-genital dysplasia (Kallman’s syndrome)-vaginal obstruction, cryptomenorrhea, imperforate hymen-receptor abnormalities for FSH and LH-specific forms of congenital adrenal hyperplasia-Swyer syndrome (XY gonadal dysgenesis)-galactosemia-aromatase deficiency-Prader-Willi syndrome-male pseudo-hermaphroditism-other intersexed conditions

ECH 41

Page 42: Internal Medicine Topics

Secondary amenorrhea-pregnancy-anovulation-menopause-premature menopause-hypothalamic-pituitary dysfunction, including

-exercise amenorrhea-stress amenorrhea-eating disorders and weight loss (obesity, anorexia nervosa, or bulimia)

-hyperprolactinemia-polycystic ovary syndrome-Sertoli-Leydig cell tumor of ovary (arrhenoblastoma; androgen-secreting tumor, sex cord-stromal origin)-intrauterine adhesions (Asherman's Syndrome)-Cushing’s syndrome-AI hepatitis (idiopathic)-PID Fitz-Hugh-Curtis syndrome (perihepatitis)-thyroid dysfunction-hemochromatosis-drug-induced

-exercise amenorrhea has been shown to be directly attributable to a low energy availability (women who exercise at a high level do not take in enough calories to expend on their exercise as well as to maintain their normal menstrual cycles)-drug-induced amenorrhea is usually caused by hormonal contraceptive medications (esp. progestogen-only contraceptives (mini pills) or high-dose formulations such as Depo Provera)-treatment varies based on the underlying condition: surgical correction if appropriate, estrogen therapy if levels are low

Hirsutism

-common (10% of women) and usually benign. It implies hair growth in women, in a male pattern (androgenic hair) [excessive hair growth that affect both men and women is called hypertrichosis]-effects are due to androgens (testosterone; hirsutism, virilsm)-causes are familial, idiopathic or are due to androgen secretion by the ovary (eg. PCOS, ovarian cancer), the adrenal gland (eg. late-onset congenital adrenal hyperplasia, Cushing’s syndrome, adrenal cancer), or drugs (steroids)- growing evidence implicates INS in women to the development of hirsutism (obese women with INS R are at high risk of becoming hirsute) – INS-lowering treatments will lead to a reduction in hirsutism in these individuals. It is speculated that high enough concentrations of INS stim. ovarian theca cells to produce androgens-one method of evaluating hirsutism is the Ferriman-Gallwey score which is based on the amount and location of hair growth on a woman-treatment: local measures (shaving etc.); estrogens (serum sex-hormone binding globulin, free androgens; always combine with a progesterone to prevent excess risk of uterine neoplasia); anti-androgen + progestogen; clomifene is used for infertility

Hypogonadism in males

ECH 42

Page 43: Internal Medicine Topics

-hypogonadism is the failure of testes to produce testosterone, spermatozoa, or both. The testes are small, with symptoms of reduced libido, impotence, and loss of secondary sexual hair-additional symptoms may be dry skin, fatigue, muscle atrophy, polyuria, dementia-like memory loss, glc intolerance (early diabetes), Ch/lipid abnormalities, depression, anxiety, osteoporosis, psychological and relationship problems, anemia, infertility-primary hypogonadism: due to testicular failure

-local trauma, torsion, chemotherapy, radiation-post-orchitis (mumps, HIV, brucellosis, leprosy)-renal failure, liver cirrhosis (atrophy) or alcohol excess (toxic to Leydig

cells)-chromosomal abnormalities (Klinefelter’s syndrome – 47XXY; delayed

sexual development, small testes and gynecomastia)-Swyer syndrome (XY gonadal dysgenesis; no functional gonads are

present to induce puberty in an externally female person with karyotype XY)-varicocele (can hormone production)-androgen insensitivity syndrome (mutations of gene encoding the

androgen R)-secondary hypogonadism: due to gonadotropins (LH, FSH)

-hypopituitarism-Kallman’s syndrome-systemic illness-Laurence-Moon-Biedl syndrome (ciliopathic disorder), Prader-Willi

syndrome- anabolic steroids-induced hypogonadism (ASIH; neg. feedback reduces

pituitary production of gonadotropins)-Dx: blood test measuring testosterone level (normal = 298-1098 ng/dl); measurement of LH/FSH can distinguish primary from secondary hypogonadism (/ in the latter)-treatment: HRT (testosterone; transdermal, injectable, buccal)

Acid-base disordersNormal values (pathophysiology department)pH: 7.38-7.42pCO2 (art.) 35-45 mmHgHCO3

-: 22-26 mMBB: 42-46 mM (48) (buffer base; all bases that can accept a proton in the blood: phosphate, Hb etc.)

BE: 02.5BB & BE are pure metabolic parametersAG: 20 (10-20) mM (>20 is always pathological)

-pulmonary compensation of disorders of metabolic origin is very rapid (secs-mins) if there is no underlying lung disease (ex.: Kussmaul breathing), renal compensation needs 2-3 days to be total

Respiratory acidosis: pCO2 >45 mmHg

ECH 43

Page 44: Internal Medicine Topics

-hypoventilation due to obstruction: COPD, asthma, pulmonary embolism, physical-depression of CNS resp. center: sleeping pills, morphine-failure of CNS control: bleeding, inflammation, thrombosis-thoracic cage disorders/musculature: kyphoscoliosis, spinal cord injury, myasthenia gravis, Guillan-Barré syndrome, polyomyelitis -compensation and BB: pCO2 + H2O H2CO3 H+ + HCO3

- H+ binds to hemoglobin, so we “win” 1 HCO3

-, “loose” 1 Hb BB doesn’t changeBefore compensation After compensation

pH (closer to normal)pCO2 aHCO3

- stHCO3

- BB BE (+)

Respiratory alkalosis: pCO2 <35 mmHg-hyperventilation: stroke, subarachnoid hemorrhage, hysteria, anxiety, pregnancy (progesterone), temperature, sepsis, thyrotoxicosis, CNS stimulants (amphetamine derivatives), pulmonary emboli (reflex), pulmonary edema, ventilation

Before compensation After compensationpH (closer to normal)pCO2 aHCO3

- stHCO3

- BB BE (-)

Metabolic acidosis-acid overproduction (AG): lactic acid (shock, infection, hypoxia), urate (renal failure), ketones (DM, alcohol, starvation), drugs/toxins (salicylates, biguanides, ethanol, methanol, acid poisoning)-base loss (AG; drop in HCO3

- is compensated for almost completely by Cl-): renal tubular acidosis, diarrhea, ileus, drugs (acetazolamide), chr. pyelonephritis, hyperparathyroidism, Addison’s disease, pancreatic or biliary fistulae, ammonium chloride ingestion -compensation should occur instantly

pH pCO2 aHCO3

- BB BE (-)Se K+ Total K+ *

*due to urination

Metabolic alkalosis

ECH 44

Page 45: Internal Medicine Topics

-severe vomiting (pyloric stenosis), K+ depletion (diuretics), burns, ingestion of base, inappropriate treatment of acidosis (iatrogenic), mineralocorticoids (Conn’s sy., Cushing’s sy.)

pH pCO2 aHCO3

- BB BE (+)

Volume disorders

Dehydration-symptoms: fatigue, xerostomia, dry tongue, dry skin, skin turgor, muscle cramps, fever (esp. in infants), headache, BP, Babinski reflex-prevention, treatment: sufficient water intake, shade, appropriate clothing; rehydration, electrolyte replacement-severe water loss → brain dehydration → small blood vessel rupture → cerebral hemorrhage death-chronic dehydration → synthesis of osmotically active substances in the brain. If rehydration is too fast → brain edema death

-isotonic: 300 mOsm/L; caused by chr. vomiting, chr. diarrhea, blood loss; no fluid movement bw. IC & EC space; polydipsia

-hypertonic: >330 mOsm/L; caused by excessive sweating, DI (central/nephrogenic), insufficient water intake; IC volume loss (EC), measured through MCV the smaller the MCV is, the more severe is the dehydration; extreme polydipsia

-hypotonic: <270 mOsm/L; caused by vomiting, burns, diarrhea, diuretic therapy (iatrogenic), Addison’s disease (aldosterone H2O & Na+ retention, K+ reabsorption), edema production; MCV, neurons also swell brain edema; polydipsia; complication: hyperkalemia ECG abnormalities (QT interval, T wave)

Hyperhydration/water intoxication-symptoms: polyuria, skin turgor, CVP-treatment: diuretics; IV saline to restore electrolyte levels; ADH receptor antagonists-Se[Na+]↓ brain edema. Acute 120 mM Se[Na+] can be lethal, but by chronic development it can be symptom-free!

-isotonic: 300 mOsm/L caused by HF, kidney failure, iatrogenic; normal MCV

-hypertonic: >330 mOsm/L; caused by Conn’s syndrome, Cushing’s syndrome (cortisol bind to mineralocorticoid Rs)

-hypotonic: <270 mOsm/L; caused by polydipsia

Osmolality disorders

ECH 45

Page 46: Internal Medicine Topics

-plasma osmolality is a measure of the concentration of substances such as sodium, chloride, potassium, urea, glucose, and other ions in blood. It is calculated as the osmoles of solute per kilogram of solvent (normal = 280-300 mOsm/L; normal urine osmolality = 350-1000 mOsm/L)-estimation: 2(Na+ + K+) + urea + glc; if measured osmolality is greater than this (ie. an osmolal gap >10 mM), consider DM, high blood ethanol, methanol, mannitol, or ethylene glycol

-osmolality can be measured with an osmometer. It works on the method of depression of freezing point-in normal individuals, increased osmolality in the blood will stimulate secretion of ADH. This will result in increased water reabsorption, more concentrated urine, and less concentrated blood plasma. A low serum osmolality will suppress the release of ADH, resulting in decreased water reabsorption and more concentrated plasma

Causes of altered serum osmolality:-increased osmolality

-hypernatremia-hyperglycemia-inc. BUN (renal failure)-toxic substances in blood: ethanol, methanol, ethylene glycol, acetone-dehydration-diabetes insipidus

-decreased osmolality-SIADH-loss of sodium in excess to water

Diabetes insipidus-def. of ADH-central DI: tumors, surgery, trauma (destruction of hypothalamic nuclei, pituitary stalk, or neurohypophysis)-nephrogenic DI: renal diseases, ADH-unresponsive kidney, lithium-characterized by polyuria (5-15 L/day). Urine is hypoosmotic (<290mOsm/kg; specific gravity <1.010). Loss of water results in mild hypernatremia and inc. serum osmolality (290-295 mOsm/kg) polydipsia

-DDx: diabetic polyuria (hyperosmotic urine, hyperglycemia, glucosuria); psychogenic polydipsia/polyuria (excessive water intake, excretion of large quantities of hypotonic urine, with normal serum osmolality and ADH concentration)-Dx: water deprivation test and/or injection of desmopressin (ADH analogue)

-stage 1: fluid deprivation (0-8h), for Dx of DI. Start at 08.00-empty bladder, then no drinks and only dry food-weigh hourly. If >3% weight lost during test, order urgent serum

osmolality. If >300mOsm/kg, proceed to stage 2-collect urine every 2h; measure volume and osmolality-venous sample for osmolality every 4h-stop test after 8h if urine osmolality >600 mOsm/kg (ie. normal)

-stage 2: differentiate central from nephrogenic DI-proceed if urine still dilute (<600 mOsm/kg)

ECH 46

Page 47: Internal Medicine Topics

-give desmopressin 20g intranasally. Water can be drunk now-measure urine osmolality hourly for the next 4h

Normal Urine osmolality >600mOsm/kg in stage 1

Primary polydipsia Urine concentrates, but less than normal (>400-600mOsm/kg)

Central DI Urine osmolality increases to >600mOsm/kg after desmopressin

Nephrogenic DI No increase in urine osmolality after desmopressin

SIADH (syndrome of inappropriate ADH secretion)-excess ADH in spite of low plasma osmolality-causes of SIADH:

-tumors: cc. of lung, pancreas, prostate; lymphoma; Ewing’s sarcoma-lung diseases: COPD, pneumonia, TB-brain diseases: encephalitis, meningitis, brain abscess, brain tumors,

subdural or subarachnoid hemorrhage, Guillan-Barré syndrome-drugs: antidepressants, diuretics, cytotoxic drugs, narcotics

-an excess of ADH results in retention of water and expansion of plasma volume. Serum osmolality is low (<260mOsm/kg), sodium concentration is low (<125mM), urine is hypertonic (>500mOsm/kg) and with sodium (>20mM)-most pt.’s are asymptomatic, typically there is no peripheral edema or any evidence of dehydration

Vitamin deficiencies and hypervitaminoses

-water-soluble vitamins (B complex vitamins; B1 – thiamine, B2 – riboflavin, B3 – niacin, B6 – pyridoxine, B12 – cobalamine + folic acid + vit. C –ascorbic acid) are not stored in the body. Regular intake is essential, except for vit. B12 (stored in the liver). Toxicity from excessive intake is rare excess vitamin is excreted in the urine-dietary sources:

-B complex vitamins (except vit. B12): whole grain cereals, green leafy vegetables, fish, meat, dairy foods

-vitamin B12: food of animal origin (intestinal bacteria in animals synthesize B12)

-folic acid: leafy vegetables, cereals, fruits, animal products-vit. C: fruits (esp. citrus fruits, tomatoes), vegetables, meats, milk

-clinical manifestations of vit. deficiencies are often shared. In B complex vitamins, deficiencies are often marked by glossitis, dermatitis, diarrhea

-fat-soluble vitamins (A, D, E, K) are stored in the body, so deficiency results from malnutrition and malabsorption syndromes, pancreatic exocrine insufficiency, or biliary obstruction (all are associated with poor abs. of fats). Excess intake (hypervitaminosis) with resultant toxicity may occur (esp. with vitamins A and D)

ECH 47

Page 48: Internal Medicine Topics

-vit. B1 def.: associated with severe malnutrition (alcoholism, fad diets). Thiamine is important for carbohydrate and AA intermediary metabolism

-dry beriberi: peripheral neuropathy atrophy of muscles in extremities-wet beriberi: marked by high-output cardiac failure, dilated

cardiomyopathy, general edema; results from peripheral dilation of arterioles and capillaries inc. AV shunting, hypovolemia, cardiac dilation

-Wernicke-Korsakoff syndrome: thiamine def. + alcoholism; degenerative changes in brain stem and diencephalon, hemorrhagic lesions of gray matter and mamillary bodies confusion, ataxia, ophthalmoplegia (Wernicke triad), memory loss, confabulation

-Rx: thiamine is available as IV, oral, IM preparations

-vit. B2 def.: rare, because riboflavin is almost always added to commercially prepared bread and cereals – occurs in alcoholics, fad dieters, elderly, ppl with chronic debilitating diseases; cheilosis, glossitis, corneal vascularization and seborrheic dermatitis of the face, scrotum, or vulva is characteristic. Vit. B is a component of FAD and FMN essential for redox processes

-vit. B3 def.: develops only when diet lacks both niacin and tryptophan (trp niacin). May occur in carcinoid syndrome and anti-TB drugs (isoniazid). Endemic in China and Africa. Niacin is a component of NAD/NADP = essential to glycolysis, TCA etc. Deficiency manifests clinically as pellagra (dementia, dermatitis, diarrhea – the three Ds)

-Rx: education, electrolyte replacement, nicotinamide 100 mg/4h PO

-vit B6 def.: may cause convulsions in infants (dec. activity of pyridoxal-dependent glutamate decarboxylase deficient prod. of GABA). Pyridoxine is required for transamination, porphyrin synth., niacin synth. (from trp). Clinical manifestations are similar to those of vit. B2 deficiency. Pyridoxine def. is uncommon, but it may occur in

-chr. alcoholism-association with drugs (isonicotinic acid hydrazide – antiTB)-homocystinuria, pyridoxine-responsive anemia (inc. need for pyridoxine)

-vit. B12 def.: daily requirement = 1g/day; liver stores 1 mg; usually caused by malabsorption, may also occur in strict vegetarians (not found in alcoholism). Malabsorption diseases causing vit. B12 def. include atrophic gastritis, pernicious anemia (lack of IF), Crohn’s d., blind loop syndrome, fish tapeworm infestation. Can also occur secondary to gastrectomy or ileal resection. Leads to marked reduction in DNA replication and cell division – cobalamine is needed for folate synth. and activation of THF, which in turn is required for conversion of dUMP dTMP megaloblastic anemia with neurologic dysfunction

-features: symptoms of anemia, combination of pallor and mild jaundice, glossitis, angular cheilosis, irritability, depression, psychosis, paresthesia, peripheral neuropathy, subacute combined degeneration of SC-Schilling test: determines whether low B12 is due to malabsorption from terminal ileum or due to lack of IF – by comparing proportion of oral dose (1g) of radioactive B12 absorbed and then excreted in urine, with and without the concurrent administration of IF (blood is saturated by an IM dose of 100g B12

ECH 48

Page 49: Internal Medicine Topics

first). If IF enhances absorption urine B12, lack of IF (pernicious anemia) is the likely cause

-Rx: hydroxycobalamine is available as IM and oral preparations

-folic acid def.: daily requirement = 50g; def. is usually of dietary origin, often occurs in alcoholics and fad dieters. Can be secondary to malabs. (celiac d., tropical sprue, jejunal resection), inc. demand of folate (pregnancy, lactation, hemolytic anemia) or cancer chemotherapy (folic acid antagonists) megaloblastic anemia without neurologic changes

-vit. C def.: due to lack of vit. C in diet; ascorbic acid is needed for pro and lys hydroxylation, DA hydroxylation in NE synthesis

-defective formation of mesenchymal tissue and osteoid matrix due to impaired synthesis of hydroxyproline and hydroxylysine (vit. C is cofactor)

-defective collagen fibrillogenesis impaired wound healing, fragile capillaries ( abnormal bleeding)

-scurvy: muscle, joint, and bone pain; bleeding gums; subperiosteal hemorrhage, perifolicular petechial hemorrhages. Bone changes are secondary to defective osteoid matrix formation

-Rx: ascorbic acid ≥250mg/24h PO

-vit. A def.: vitamin A is a term for a retinoids with similar activities, essential for the maintenance of mucus-secreting epithelium. It is also important in glycoprotein synthesis. Retinol (a derivative) is a component of the visual pigment rhodopsin. Def. can be caused by dietary deficiency or fat malabsorption and manifest as night blindness, squa. metaplasia of trachea, bronchi, renal pelvis, conjunctivae, and tear ducts, and ocular abnormalities (Bitôt’s spots – oval or triangular spots on conjunctivae, xerophthalmia, blindness, keratomalacia (corneal softening))

-Rx: vit. A 200 000 IU PO, repeat in 24h and a week later

-hypervitaminosis A: due to excessive intake alopecia, hepatocellular damage, bone changes

-vit. D def.: vit. D (calciferol) is synthesized in the skin by UV light from the precursor 7-dehydrocholesterol. It promotes intestinal calcium and phosphorus absorption enhanced bone calcification. Def. manifest as rickets in children and as osteomalacia in adults, both due to deficient calcification of osteoid matrix. Can be caused by

-malnutrition-intestinal malabsorption-inadequate exposure to sunlight-liver d. (impaired conversion of vit. D to 25-hydroxyl form, precursor of

calcitriol)-renal d. (incomplete synthesis)-vitamin D-resistant rickets due to hereditary renal 1-hydroxylase def.

(impaired calcium abs., inc. PTH activity)

ECH 49

Page 50: Internal Medicine Topics

-hypervitaminosis D: manifest in children by growth retardation and in adults by hypercalciuria, nephrocalcinosis, renal calculi

-vit. E def.: rare; thought to result in hemolysis and neurologic dysfunction. Vit. E (-tocopherol) is an antioxidant, metabolic function is maintenance of cell membranes

-vit. K def.: vit. K is essential for carboxylation of glytamyl residues in the synthesis of clotting factors II, VII, IX, and X, and of protein C. There are no known clinical manifestations of vit. K. Def. results from fat malabsorption or alterations in the intestinal flora caused by antibiotics. Characterized by hemorrhagic diatheses (prolonged PT and aPTT) – may manifest as hemorrhagic disease of the newborn

-Rx: 10mg vit. K IV or FFP for acute hemorrhage

Sources:-own notes-lectures-Longmore et al.: Oxford handbook of clinical medicine (OHCM), 7th ed. -Kumar et al.: Robbins Basic Pathology, 8th ed.-Damjanov: Pathophysiology, 1st ed.-http://www.endotext.org/neuroendo/neuroendo11b/ch01s03.html (and some other websites - I forgot to write down the links!)

ECH 50